The Way is Shut

When I first approached the topic of societal energy in 2004, I became aware for the first time that our energy future was not in the bag, and proceeded to explore alternative after alternative to judge the viability and potential pitfalls of various options. I have retraced my steps in Do the Math posts, exposing the scales at which different energy sources might contribute, and the practical complexities involved. My spooky campfire version of the story, a la Tolkien: The Way is Shut.

Alright, I’m overstating things a bit. The good news is that there do exist energy flows and sources that qualify as abundant or at least potent. However, many of the alternatives represent ways to produce electricity, which applies only to about one-third of our current energy demand. The immediate threat is therefore the short term liquid fuels crunch we will see when the global petroleum decline commences within the decade.

In this post, I will reflect on the lessons we learn after having characterized the various alternatives to fossil fuels. There will still be some tidying-up to do on energy alternatives not treated thus far, but by and large the nature of content on Do the Math is about to pivot toward addressing the question “What can we do now?” In some sense, a common thread so far has been: “easier said than done,” or “don’t count on that technology saving our bacon.” I’ve closed all the exits to get your attention. We’re boxed in. Okay, the exits aren’t really closed: they’re just not as wide open as they would need to be for me to be complacent. So now we’ll start looking at ways to nose out of our box in a safe and satisfying way.

Summary Matrix

A few weeks back, I organized assessments of fossil fuel alternatives into a scoring matrix to provide—at a glance—a sense for the pluses and minuses of each option. We saw from this exercise that most alternatives are inferior replacements for fossil fuels in one way or another (although superior in terms of carbon dioxide emission). We also saw that transportation will be the hard part. Here is a repeat of the matrix (click to enlarge). See the post for details and commentary, or click here for the colorblind-friendly version.

My Biggest Surprise

The raft of comments following the summary post highlighted something that for me was unexpected. Even though my primary messages were that alternatives fail to stack up to the familiar benefits of fossil fuels, and that transportation is particularly challenging to cover, I seem to have stirred a hornets’ nest in ranking nuclear options well below solar. I saw many comments arguing for nuclear’s superiority over solar energy. I never appreciated the apparent rivalry. So that was interesting. It seemed that nuclear enthusiasts were too perturbed by my admittedly simplistic ranking scheme to appreciate the larger points. I’ll bet this tendency extends to enthusiasts of all stripes, and imagine that had I inverted the rankings, the solar enthusiasts would have come out swinging.

To some degree, this is worrisome to me. If we see alternative energy as a competition, we may all lose. Let’s fight the fossil fuels, not one another; get started transforming our future; let’s clamor for funding for all of our viable alternative approaches—each offering unique advantages (and, yes, disadvantages).

Your Nearest Exit May Be Behind You

How do we respond to our looming energy predicament?

Picture yourself in an jet airliner. The captain makes an announcement that the airplane is going to make an emergency landing (something about running out of fossil fuels), but that it should be a smooth glide to a landing on the flat prairie below. The professional flight attendants calmly explain how the exit doors and slides work. A quick calculation in your Do-the-Math head says that eight functioning exits and 200 passengers means 25 passengers per exit, and complete exodus within a minute if passengers can get out at one every two seconds. Not bad.

Knowledge of the exits brings peace of mind. If the landing is indeed smooth, no problem. But if the meeting with the ground is abrupt, what looked to be good options during the calm order preceding the crash no longer seem viable. The door ahead on the left has been wedged and is inoperative. The door opposite has fire immediately outside. You can’t see well enough through the smoke to tell if the doors ahead of those two are viable, but the panicked voices in that direction do not bode well. It seems there must be some door near the back that’s working, given the kinds of shouts you hear in that direction. But the press of people and the various stupid decisions you witness complicate matters (why is that idiot trying to carry out his suitcase?!—the strap is caught on the seat arm and causing a blockage in the aisle).

Analogies are never perfect, and this one is a touch on the over-dramatic side. Obviously, the “exits” here are solar, wind, nuclear fission, hydroelectricity, geothermal, biofuels, wave, tidal, etc. The fact that we can rattle off a lot of names for alternative energy sources comforts some people enough to skip the precaution of guaranteeing the viability of those exits when they are needed. Indeed, the sheer numbers are ample for solar, fission (especially breeding from thorium), ocean thermal, and nuclear fusion for that matter.

Yet the numbers are only part of the story. On top of this is a layer of practicality. I don’t care how much deuterium ordinary water contains, if fusion is not technically viable when we need a large-scale rescue. Solar and wind are intermittent and require storage solutions that we seem not to have at adequate scale (wishful thinking suggests they may complement each other’s intermittency at scale).  Ocean thermal is hampered by a long list of practical/logistical impediments. A next-generation nuclear fission push is among the most promising, but it’s not here today—and until it is proven to be effective and economical, wisdom suggests that we not count it as certain. After all this, we are still left with a mismatch between the type of energy we need (for transportation) and the type we can most easily make (electricity).

When the descent stage of petroleum hits and production drops by several percent per year, the economic shocks and global reaction will be significant, and we will be scrambling to find our exits—only then to realize that nothing is as easy as it seemed during times of surplus, and that all new infrastructure efforts require the very energy that is in short supply (the Energy Trap). It’s true that shortage of one form of energy does not mean shortage of all types of energy. A liquid fuels shortage won’t directly translate into electricity shortage, for instance. But virtually every facet of our modern society requires our transportation capabilities to remain intact. Without that, virtually everything becomes hard.

In much the same way, kidney failure does not directly impact the function of hands or legs—yet you are unlikely to find someone suffering from kidney failure out in the garden yanking up weeds. Systemic failure is a tough nut.

As in the airplane, the nearest exit may be behind us: meaning that we may do well to consider a simpler lifestyle that may strike some as moving backwards.

Conservatism and Asymmetric Risk

I will disclose that politically I am a registered Independent with both liberal and conservative sympathies, having voted for presidential candidates across the political spectrum. But when I say conservative, I mean it in the traditional sense—not by today’s ideological definition of the term. Conservatism to me means taking a low-risk approach to our future: not gambling the prosperity of future generations on uncertainties of today. Conservatism to me means conserving resources, conserving energy, balancing budgets (raising taxes is certainly a viable option in my book), and having a well-thought plan for the future that also acknowledges uncertainties. I have been tempted to make a bumper sticker asking ideologically-identified conservatives: “What have you conserved for me lately?” But alas, I don’t drive my truck enough to make the effort worth my time.

I also find that science breeds traditional conservatism into its practitioners: we dare not publish wild claims that are not backed up by careful research, experimental confirmation, repeatability, etc. It’s not a perfect institution, but—holy cow—does it work surprisingly well given that imperfect humans are behind it all!

So my tendency to resist the “hopium” of future energy technologies as our salvation is a reflection of this conservatism. Imagine that I issue the following statement:

“The near future introduces monumental resource challenges to which I see no obvious solution. Unless we devote an unprecedented amount of attention to this problem, we risk overshoot, collapse, and loss of all the knowledge we’ve worked so hard to acquire.”

To which someone counters with:

“This view is unjustified. We have plenty of viable options in unconventional fossil fuels, thorium, solar, space resources, ultimately fusion, and other ideas to boot. And let’s not forget the human brain: an inexhaustible resource. We’ll always figure our way out of a fix—once the price is right—and the future will be more amazing than you can imagine. You’re only setting yourself up to be as much a laughingstock as Malthus.”

Okay. I am not at all unfamiliar with this reaction—frequently represented in Do the Math comments. First, I firmly believe that Malthus will ultimately be proven right that growth collides with finite resources so that growth must stop. His timing was off because he did not see fossil fuels coming, and I could likewise be accused of not seeing the next big wave of energy that will wash over us and “kick the ladder” of fossil fuels out from under us—a compelling notion, to be sure. But mathematical and thermodynamic arguments alone portend the end of growth in a finite system. Scientists will not disagree. The very first Do the Math posts address this. Confined to Earth, we boil ourselves in a little over 400 years on a 2% energy growth trajectory.

That said, in the nearer term (this century), I might be foolish to worry so much about our energy future on our current trajectory. We may develop suitable substitutes to fossil fuels in a smooth and timely way (if I can ignore concerns over the fact that we can’t seem to take care of our gerbil, while pining for a pony).

So here’s the thing. What if I’m wrong to be worried? Or what if my imaginary critic is wrong? Which is worse? If I advocate a path of restraint and careful transition to a possibly lower-energy future and I am ultimately shown to be wrong about the limits we face, what’s the damage? In this scenario, we’ve stabilized our system into something approximating sustainability. If we learn later that we have more resources available, we can make the choice to spend them profligately, use them sparingly, or ignore them. But we do so from a position of stability. If, on the other hand, the critic convinces us that the future is up, up, up, and we don’t take resource limits seriously then their being wrong is disastrous because we charge into overshoot, overextension, hit resource limits hard, and run a serious risk of societal collapse.

It’s a classic asymmetric risk problem. Believe me when I say that I’m happy to be wrong about the seriousness of our predicament. And if I thought that the scenario I paint had less than a 1% chance of transpiring, I could go along with the dismissals and find other ways to spend my time. I happen to perceive the likelihood of failure to be significantly higher than this. If true, then prudence demands that we assume that pursuit of business-as-usual will not prevent collapse—even if this view is ultimately understood to be an overreaction. For similar reasons, we buy fire insurance and flood insurance for our homes despite very low odds of disaster.

One of my most compelling pieces of empirical evidence that we are at risk of collapse is the degree to which most folks I run into can’t entertain collapse as a serious possibility. In fact, I usually feel like such a kook even suggesting it that I rarely put it “out there” in casual interactions—even when relevant to the conversation. Usually, time does not allow conveyance of the extensive background that is necessary to lend credibility to such a statement. Look how many words I have spilled (about 100,000) in Do the Math to justify my concerns.

In the face of serious resource challenges, I would hope to see more attention given to a high-stakes loss/collapse scenario. The opposite of attention is inattention, and that’s not going to help mitigate the problem.

Differentiating Opinion from Wisdom

I only fell into this “limits” camp because practically every time I performed quantitative analyses on this, that, or the other alternative energy proposal, I came up disappointed. I really did want the pleasure of personal discovery that we have an obvious path forward. I am delighted by the abundance of solar energy input to the planet. I am reassured by the vastness of thermal energy in the oceans and crust. I am tentatively excited about the vast energy represented by uranium in the oceans and by thorium using functional molten salt reactors. I truly do see these as positive lights in the darkness.

Yet over and over, quantitative analysis knocks out many of the “exciting” ideas we hear about in the sensationalized media world. Already, this is a damaging blow to our collective perception that solutions abound. But the quantitative analysis does turn up a few gems that have the numbers behind them. Is this enough to allay my worries? Obviously not, or I would be merrily spending my time in ways other than writing a time-consuming blog on top of a very demanding research/teaching job.

Why am I stubbornly unconvinced by my own computations that show a few exit doorways lit up through the smoke? All I can offer is that I draw on my hunches as an experimental scientist. In that capacity, I build stuff that works. To do this, I have to be practical. I have to have a “spidey-sense” of what things are likely to succeed or fail—sometimes folding in political factors as well. I clearly don’t find all ideas to be impractical, or I would never have rushed headlong into the many projects that I have pursued.

I have on occasion found myself challenged about specific choices I made in designing an instrument by folks without instrument-building experience. Wouldn’t it have been better to choose a different size/value for X system? Theoretically, sure—if only narrowly considering one particular aspect. But I had to balance that choice against ten other competing considerations that are often difficult to articulate or even recall on the spot. Then I remember that the person asking the question has not personally experienced the compromises of building a complex instrument that works.

Likewise in imagining our future path. A complex, interrelating series of considerations will steer us one way or another. My hunch is that human nature, political realities, economics (including economic hardship) combined with technical shortcomings of alternatives will get in the way of our shiny future. I would like to be convinced that this isn’t the case so I can stop worrying and go full-force on my experimental physics career, but the arguments for why things will be alright often strike me as narrow or simplistic. “It’s obvious: we’ll go to space where resources are unlimited.” “You’re forgetting something very important: human ingenuity—an unlimited resource.” “More sun hits the Earth in an hour than we use in a year: it’s obvious we’ll solve this problem.” “We have enough fuel sitting in nuclear waste pools to power us for millennia.” “Peak oil will not be a problem because we have tons more hydrocarbons in the ground beyond conventional petroleum.” You get the picture: a key idea that will make everything work out. It has the same ring as “Home prices in San Diego can never go down because it is such a desirable place to live,” which I ignored in 2005 in favor of data and more complex analyses.

Any argument/rebuttal that starts with “It’s simple,” or “It’s obvious,” or something along those lines is more likely to fall in the foolish camp than the wisdom camp. Perhaps it is my experience dealing with multi-faceted complexity that draws me toward nuance and hedging and makes me wary of simple black-and-white assessments that profess certainty about the future. In the end, I have little choice but to trust my instincts.

My “hunch” tells me that we work in an imperfect world full of irrational reactions, uneducated citizens, dysfunctional politics, competitive nation-states willing to wage war, ruthless extrapolation based on our fossil fuel bonanza, and simple, stupid inertia. The problem transcends narrow academic and quantitative analysis—although I treasure the math and believe it should always be a vital part of any analysis. Obviously, quantitative failure trumps other concerns, although quantitative success is only the first step of many in a full assessment of viability. And it is against these myriad subjective factors that I find myself down-weighting the strictly quantitative conclusions, finding plenty of room to worry.

What’s at Stake

Talking with a friend, I cast our impending collision between finite resources and a growth trajectory as being akin to driving full-speed toward a cliff. If you pass a sign that says “Cliff Ahead: Road Ends,” why would you possibly decide to keep barreling ahead? His answer: “I’ve seen that sign 20 times already. Why should I believe it this time?” Yeah. Why should he? I’m totally sympathetic to this reaction. Had I not been stymied time after time by my own quantitative analyses and consideration of practical challenges, I might well have the same attitude.

But to me, the sign has achieved a credible status. Is it right? I can’t say for sure. But I judge that we should heed the warning and adjust our behaviors accordingly. Slow down. Make a turn toward sustainability, even if it’s rough at first. Do it while we can. Be conservative. Creep up cautiously toward the purported edge and see if there is any truth to it (maybe already traveling sideways on a sustainable tangent).

What hit home for me personally is the notion that a worst-case collapse of civilization (not unknown to history, let us recall) would be damaging to the thing I hold dearest: our accumulated knowledge of how the world works—science. Science is a luxury of highly functional societies. It is no coincidence that scientific advance is most rapid in this day and age when surplus energy is at its peak. How many computer records, tapes, CDROMs, etc., risk destruction or degradation in a collapse—even if it lasts only a century. In the more dismal collapse scenarios, how many science journals are burned for warmth? (It’s fairly certain that volumes of the Astrophysical Journal will disappear from the library before Adventures of Huckleberry Finn is sacrificed: ApJ does not make for entertaining fireside reading.)

It was always implicit for me that work invested into science will stand for all time. But the notion that my contribution to science—however incremental—may be irrevocably lost has taken some of the appeal away, I must admit. It would seem prudent, then, for scientists to devote time and talent toward our impending energy challenges. The first step is to convince people that we must swing our attention hard-over toward understanding exactly how we wean ourselves off of the fossil fuel lifeblood of our society. Either we figure it out or Mother Nature will do it for us. I for one want to fight to keep humanity’s most impressive achievements intact and understood!

I’m the type of person who runs for the just-passed bus to try and catch it at the next stop, or for the airplane that is supposed to depart in ten minutes; as long as I perceive there is some chance, I do not want to accept defeat. I have a similar urge when it comes to our future challenge: this predicament requires all-out commitment. The problem is, commitment on an individual scale does not amount to much. That’s why I started Do the Math: to convey my sense of just how challenging our future will be, so that we might increase the chances of some collective action that can make a difference. My path started with hope, but was largely supplanted by fear. I apologize for resorting to similar tactics for my audience, but fear sure made me change my behaviors and expectations, and it may turn out to be an effective tool for us all. So…Boo!

Next week, I’ll seek to provide some balance with my picture of hope.

Hits: 6071

203 thoughts on “The Way is Shut

  1. Somehow, I don’t think you’re on the short list to be President Santorum’s science advisor.

    • I’m sure I would not make anyone’s short list—or long list for that matter. My advocacy of pulling back from economic growth is political suicide. Which is why we need to change perceptions and attitudes among the people: politicians are a dead end until then.

      • Have you looked at the Green Party? Their current platform includes the following statement: “Economic growth, as gauged by increasing Gross Domestic Product (GDP), is a dangerous and anachronistic American goal. The most viable and sustainable alternative is a steady-state economy.”

        True, they don’t have the proverbial snowball’s chance in hell of actually winning the next election, but a protest vote is not necessarily a vote wasted.

    • I believe Steven Chu has been accused repeatedly of selling us “technium” – the liquid fuel version of hopium that will fill our gas tanks with human ingenuity and combust technical innovation. Kunstler, bless his curmudgeonly ways, has choice words for those that confuse the reality of billions of tons of hydrocarbons with our ability to refine gadgets. In short, Tom Murphy might not make Obama’s list either.

      • Steven Chu and many other physicists today don’t have a background in energy [Chu worked in Bell labs], they were born in an energy rich world that enabled them to groom in their professions. The larger numbers of scientists and engineers who were involved in energy were in the times when fossil fuels were being developed and harnessed, they have passed and now the lot that is left is mostly the one that has been bathing in energy, they were born into the party, their focus has not been energy since it was a problem that wouldn’t show up until after a couple of generations of fossil fuel burning. Many sharp minds in science and engineering turned to non-energy areas that were more exciting like computers and electronics and have very little understanding of energy problems, these industries were enabled largely due to energy surpluses. Ironically, their successes may be the very bottleneck behind their inability to grasp the energy reality, they have perhaps unconsciously developed a kind of technoreligiosity as a plug in for energy. Many scientists/technologists today have very little core understanding of energy, even though most are aware of the law of conservation of energy. Most of the disciplines remain disconnected, physicists continue to ignore biology/environment/sociology etc and vice versa. On top of that, the fradulent profession of business/economics [as practiced today] has polluted real efforts. Technoreligious thinking is blurring too much of the energy realities and blogs like this one are helping uncover some inconvenient truths.

        The engineers and scientists need to turn their attention to the problem of energy and environment and sustainability in general, this needs to happen globally on a large scale.

        To address the challenges we face in a manner consistent with a civilized social order, there needs to be a global awakening [some signs are there] and people of all kinds have to unite in purpose and direction towards sustainability.

        • I can confirm that most physicists I deal with are not energy-aware. Usually it’s those who have taught classes on energy/environment, which is a definite minority. As with most other people, there is no need to be aware. The light switch makes the light go on, and there is always gas at the pump (albeit increasingly expensive).

          • Wow. I had never even considered this possibility. I mean, isn’t conservation of energy and a working knowledge of thermodynamics enough to conclude, from first principles, that while we might well have *enough* energy at a given time, consumption cannot grow without limits. It should also be obvious that energy density matters.

            That’s not even common sense of the “everything important is hard to build” type found in day-to-day experimental physics.

            But then, I never understood all the knee-jerk Malthus and Club of Rome bashing either. One can argue about the numbers – and should – but the response at the time, and to this day, was that of faithful facing sacrilege. For some reason, the generations alive do not seem to tolerate the concept of limits at all. Goes hand in hand with the inability to deal with mortality, maybe?

        • Steven Chu is far, far, far smarter than you people seem to believe. And not just in the laureate-winning department, but in the far more important team building and team leading and political navigation fields.

          He can Do The Math just fine. He also knows when to sugar coat and when to Keep His Mount Shut.

          I’d last about three minutes in his position, and not because I’m a less capable physicist. The same is true of everybody else reading this, guaranteed.

          The most important thing he can possibly do for the world is to not get fired and not get his boss fired. Coming right out with the naked truth — “we’re completely [screwed], and everybody’s version of The American [economic] Dream is impossible, as any idiot with primary school mathematical ability can verify” — would be the best way to make a weetched political situation even worse.

          Yes, profound expertise in one technical direction doesn’t guarantee freedom from yawning blind spots or freedom from neanderthal politics (Teller, Dyson, a thousand others), but give the man some credit! Listen to exactly what he says, read between the lines, and do the math.

          • I’ll back this up. Am I disappointed in the limitations to what is politically viable? Absolutely. But I’m glad a super-smart physicist is in the position of Sec. of Energy, doing what I imagine to be the best job that our population will allow. It’s peoples attitudes that need to be shaken up and changed. Politicians (and their appointees) cannot afford to be leaders on unpopular topics (energy reduction, for instance), or they get voted out of office.

            But I do read between the lines, and think these guys do have an inkling about the fundamental problems, and are operating within the constraints imposed by us idiotic voters.

          • What would you do if you were in Chu’s position and knew the truth? Wouldn’t you tell the truth as you see it instead of waiting for the world to crash? Wouldn’t you resign in frustration and do something else when you can hardly achieve anything in this position? That would send a strong message. There is no leader today coming out in the open and telling the truth [utter stupidity of the economic growth nonsense and energy difficulty] to the American people. The only politician that openly talks about peak oil is Roscoe Bartlett [R-MD], he is a scientist. What did Chu tell last time he made public statements? We are working on electric battery technology and we will get there in a few years. He mentioned nothing about we may not get there [or the practical difficulties of transition time to these] and that we need to prepare for that possibility. The techno optimism is sickening. That is not leadership, that is cowardship. I know he is a knowledgeable man but scientists need to be leaders and come out in the open and become activists [e.g James Hansen]. Our path forward can only come by first embracing the truth fully and talking about it and fighting for it. Einstein said “The right to search for the truth implies also a duty; one must not conceal any part of what one has recognized to be the truth”. We need to be bold, such as this young man :
            http://www.youtube.com/watch?v=1X3VynNZQaQ

    • Not to go down a political path, but these reasons is why I fear politicians and what they do to a country. Since congress controls the budget and what gets done, it saddens me that they all are lawyers and none are scientist or technologist. Why do we not have a Dept of Science in the Executive branch where all the science fields are represented? Where math and ideas like this blog are actually done on the Government level and discussed with the Congress as much as National Security briefings are.

      • The grass always looks greener on the other side.

        Science is too important to be left to government, for politicians and uneducated bureaucrats, left to their own ends, are perfectly content to appoint and champion the next Lysenko, then pat themselves on the back for having done so.

        If we had a department of science, would it suffer from regulatory capture? What happens when the government can operate a Ministry of Truth because its scientists “say so”?

        When the Catholic Church (a/k/a government of the time) ran science we were forced to accept a flat Earth and intelligent design.

        We need healthy competition in science between government, academia, and private industry. Good science is a spark in a creative mind, not a product of bureaucratic procedure.

        • “When the Catholic Church (a/k/a government of the time) ran science we were forced to accept a flat Earth and intelligent design.”

          I think this paragraph is wrong in every detail.

      • Don’t worry about science in government, I heard recently that 70% of the leaders in China are engineers… or does that mean we should worry?

        • it means there is hope yet. China (and its government) has shown again and again that they are more flexible than any western country, and more than anybody in the west thought possible. Most sustainable energy generation capacity today is being installed in China. Admittedly, so are coal plants, but my point is that Chinas government has a surprisingly non-ideological approach to the whole problem, and they _do_ plan for the next 50 years, not just the next election period. Of course this is because there are no free elections anyway, but on this particular issue that seems to be plus…

      • Politicians? It’s not like politicians are some independent force. Haven’t you noticed? Our present political system, in which candidates become viable and compete for election only by first acquiring huge flows of corporate cash, is utterly bought and sold by big business and finance. Truth about energy and growth is absolutely anathema to capitalists, given that it is an obvious endgame. Hence, it is forbidden by private interests, not public ones.

        It scares me that we who are open to the science of energy reality remain so blind to the basic social science of the thing. Government bashing only serves the status quo.

  2. Well, how much of our current energy needs that are met with fossil fuels could be met with electrical inputs instead? Buses can be turned into “trackless trams” or replaced with electrical-drive streetcars, and 500-mile airline routes can be replaced with electrically-driven high-speed rail (even freight rail can be electrically-driven, if we want to do so). This is all obviously on the scale of decades, but any real solution would more or less have to be, would it not?

    • I keep hearing this stuff about the need for hi speed rail. I think that this is a dangerous move. The complexity and energy requirements of building a transit system expands exponentially as you increase the speed. Amtrak would be dramatically better if you just increased the number of trains. My significant other and I use Amtrak to visit each other across 800 miles (I live in Guelph, Ontario—she in St. Louis Missouri.) Travel would be a minor inconvenience except for the fact that we have two layovers that add 15 hours to the trip—each way. Add in a few more trains on each route, and this goes away.

      I’m afraid that society will [waste] very scarce resources building a small number of high speed trains that will end up being as energy intensive as airplanes instead of building a lot more regular trains that will be very efficient.

      Trains simply cannot compete with jets in terms of speed and people have to stop comparing the former with the latter.

      • 1) Planes not only have to go faster, they must also work to gain altitude. Unless you have some numbers, I am going to remain thoroughly skeptical that a train making a trip between cities in approximately the same time as an airplane consumes more energy.

        2) Planes vs High-speed rail: for routes of up to several hundred miles, rail competes (especially when you measure time door-to-door). And in places with thriving air markets, the introduction of rail lines (e.g., NY-DC, NY-Boston, Barcelona-Madrid) ends up eating a massive portion of the air market.

        3) Amtrak is not a functioning system (and it appears that you’ve first-hand experience with that), and I agree that better and more connected routes would be nice, but you’re also taking a train on a route that, honestly, would be better served by a plane flight. LA-San Fransisco is a route well served by rail, for example, but Ontario to St Louis? Not the best example to draw from, honestly.

        4) The Japanese and French have been running high speed rail for decades. It’s not as if it’s an untested technology.

        • The Boeing 747-400 has a takeoff mass around 400 tons, so that climbing 10 km requires 4×1010 J, which at 20% engine efficiency (guess) requires an input of 2×1011 J. At 36.6 kWh per gallon of gasoline, this takes about 1500 (U.S.) gallons of fuel.

          Meanwhile, I find that the plane gets 91 miles per U.S. gallon per passenger, which at 500 passengers means 0.18 miles per gallon straight-up.

          So flying about 275 miles (450 km) takes as much energy as the climb. But notice that the descent throttles back significantly, so some of this energy is regained. I’m going to say that climbing is not a big deal.

          • I vaguely remember that on my last flight (A380, Frankfurt-Tokyo) the pilot quoted something like 3 liters per passenger per 100km, which is about the same efficiency as a recent car with two people in it, or a less recent one with four.
            Again completely from memory: I heard high speed rail in Germany is on the order of the equivalent of 1l per 100 person kilometers, unfortunately I do not know if that is calculated well-to-wheel, or only electrical consumption. In the former case it obviously beats the car and airplane hands-down, in the latter it is about equivalent.

            But more importantly of course: It has the same or better efficiency as cars and airplanes, it can replace both in many use cases, but it does not require liquid fuels! Only electricity, which we have found is much easier to supply sustainably.

          • I’m going to say that climbing is not a big deal.

            I looked up the fuel burn figures for the 747 a while ago, and it surprised me how small the figures for climb were.

            Modern turbofan engines are upwards of 40% efficient, if memory serves. The ground-based versions used for electric generation hit the mid-40’s.

        • Sorry if I was inarticulate. What I meant to write was that hi-speed rail uses a lot more energy per mile than lower speed rail. The problem isn’t that rail is just too slow, its that people in our society have gotten a tremendously weird sense of time and convenience that makes anything less than a jet trip seem totally beyond all reason.

          And as to convenience, there used to be a train that went directly from my home town to Chicago, which would have made the trip to St. Louis very convenient. The problem, I’ve been told, is that after 9/11 the border people became so idiotic that it held up the train too much, so they stopped it. I know now that the train routinely takes 2 hours to pass customs. (They need that time for a bunch of obese customs officers to personally threaten and try to intimidate everyone on the train.) Add that to the 15 hours of layovers—-.

          Isn’t it just too bad that I’m trying to lessen my carbon footprint instead of just doing my best to guzzle jet fuel like every other moron in North America?

          • live in Chicago, had a week long trip to Duluth once and looked at Train. It was some god awful long trip that got me there at a strange time too. Beyond the 300 mile drive range that i prefer. So I had to fly to Minn, get a jumper to Duluth.

            Have also done work in west suburan Detroit, Louisville, and Cleveland. I drive those now since door to door is same or less (with air delays). A 100 mph train should be able to equal that for me.

          • “What I meant to write was that hi-speed rail uses a lot more energy per mile than lower speed rail.”

            Well, er, how much? Given the what website this is on, some figures would be appropriate … :]

            A bit of googling gives figures of about 0.12 – 0.15 MJ/seat-km for good modern HSR (specifically for the TGV duplex Paris-Lyon, and the N700 shinkansen Tokyo-Osaka). The energy consumption for low-speed rail varies; I can find figures that are lower (even sub-0.1 MJ/seat-km for very modern trains), some higher (older equipment).

            But despite HSR’s reputation for being “inefficient”, I can’t find any evidence that that’s actually true…

            In part, I suppose it’s because modern HSR is made as lightweight as possible, to reduce accleration times as well as save money, and slower trains may often use older, cheaper materials and construction, and so tend to be heavier, etc.

            But still, I see no reason to be frightened of HSR; it seems a very good candidate for routes where the distance is a significant factor — after all, HSR is much more efficient than air travel (an order of magnitude, it seems).

        • I think the difference is between practical and possible. The choice is this :
          We can restore 19th century level functionality to our rail system, enabling it to be the primium mobile of the continent. That means many more trains on many more routes at hopefully highway speeds. Expansion and electrification would cost billions.

          For the same billions, we could give ourselves a handful of high-speed corridors. Maybe. If the politics ever worked out. The Japanese and the Europeans built their systems early enough that they did not face that choice. Their high speed networks supplement excellent conventional rail infrastructure, which was never lost.

          Saying the Guelph-St.Louis route is better suited for aircraft presupposes a plentiful supply of liquid fuels with which to fuel said aircraft. That’s only true for the moment, and may not remain so permanently.

          • I think that any estimate of the energy costs that we will find comparing high speed rail and medium will consist of “booster” math. As another poster—who identified himself as an engineer specializing in public transit—commented earlier, there are enormous capital costs for upgrading a railroad bed for high speed capacity. Even if we are going to build high speed rail in the country to link cities, we will still need to build feeder systems to connect these terminals to where people want to go. In a time of limited resources it makes sense to pick the low hanging fruit first.

      • Trains can compete with air over up to say 500 mile distances; plane’s superior top speed is offset by city-edge airports vs. city-center train stations, lower security screening, and with a good system, less need to be paranoid about missing your flight. There’s also competition in cost and convenience to consider.

        As for energy, http://www.inference.phy.cam.ac.uk/withouthotair/c20/page_128.shtml has jetliners as taking 10x the energy per passenger mile as electric high speed rail. And the HSR looks to be only about twice as energetic as the ordinary rail, though he’s using fairly slow HSR (200 km/hour vs. 150, when working systems are over 300 km/h).

        Also, we’d normally expect quadratic dependence, not exponential: E=mv^2/2. Maybe cubic if you’re in a bad friction regime, though dealing with friction is exactly what long trains with steel wheels on steel rail do very well.

        • My wife and I caught the train from Edinburgh to London quite a few times when living there. It was faster, more convenient and more comfortable than flying — we could walk to the local station, and were dropped off at Kings Cross in the center of london. The trip was under 5 hours (often under 4.5 hours).
          I’ve also caught the TGV East from Paris (the very fast line), which was awesome 🙂

      • Trains can run off electricity (and they mercifully don’t have to carry the source for that electricity with them) whereas planes need liquid fuels – and not just any liquid fuels. I’m fairly sure that you couldn’t run a 747 of biodiesel, they need pure, high grade stuff that is going to dry up quickly in the fossil fuel crunch.

        If you have HSR that is electrically powered, then so long as total energy availability is taken care off, you can ignore any crisis in one particular energy source. Passenger aircraft live and die on the availability of fossil fuels, and unless you fancy flying in a nuclear powered plane, that is not going to change because other power sources simply do not have the power/weight ratio for flight.

        • I think people are missing Bill Hulet’s original point.
          Building and operating high speed rail is *far* more expensive than building and operating medium speed rail, or, as I like to call it, “express” rail.

          The capital costs for high speed are enormous. double your speed from 100 to 200mph, and the curves get 4x wider, the gradients must be 4x flatter. You can;t have any level crossing, so every road that crosses the line must be either a grade separated crossing, or else it is cut off – leading to a real “divide” in the areas through which it passes.

          Power increases 8x with the doubling of speed, so 8X the motors, brakes, current capcity of wire and switchgear, and on and on – can yous ee the $ adding up.

          And when all this is done, the train then has to charge such high prices that it is still cheaper to drive

          The objective of rail should not be to compete with air travel – it should be to compete with the car.

          80-100mph, and the ability to work, text, eat, drink, etc, is plenty fast enough for that.

          • Agreed, esp. with respect to pod and container cargo: “classic” rail first. But even so, 100mph and high speed lines are not mutually exclusive as long as the resources are there. If the latter were funded out of the resources expended on airports, in whatever places they can economically be placed, there is no reason to – eventually – have both.

            As a priority, high speed rail just serves as a political means to prevent a pervasive railroad network.

          • Another cost HSR cost point that is often missed is that, generally speaking, freight can’t use the HS tracks. Freight mass is too high to run on HS tracks. Freight could possibly run at traditional speeds on HS tracks but apparently the two (HS passenger and slow freight) are simultaneously compatible for practical reasons.

          • I assume you mean incompatible.

            I don’t think slow and fast trains mix well. Especially high frequency fast trains.

            And even at the same speed, passenger and freight may not mix well. USA gives priority to freight, who own the rails, and Amtrak typically has to *back up* in the middle of a trip, to give way for freight. Conversely, Europe favors passenger rail, and freight there is said to be rather undeveloped compared to the US. (Though the continent still has working canals.)

            Also, when US freight acquired the rails, they downgraded them from being able to support the 90-100 mph speed of old streamliners, to save on maintenance. Freight’s happy with low speeds, but even if you’re a HSR skeptic, you should grant it’d be nice for the passenger trains to go at least a bit faster than cars.

            I’m thinking freight and passenger rail probably don’t mix well, period.

          • yup.

            I live in france, and I avoid the TGV if possible, instead relying on cheaper, lower trains on short travels, the time difference being negligible anyway.

            On long travels, there’s no choice, and not only is air travel competitive with train, car travel is actually cheaper, so people favor it

      • trains cannot compete with jets in terms of speed, but jets cannot compete with trains in number of passengers carried, or tonnage moved.

        (Assume double-track rail, and trains following each other at their braking distances, and you’ll get an idea of what the theoretical upper limit is to line capacity.)

        • Trains also compete well in energy efficiency, and not just because they’re slower.

  3. The point that we will lose all our hard-earned knowledge is very rarely raised and it is something that I have pondered over a lot (to the point my own research sometimes suffers because the thought that all the knowledge you generate will be lost is not exactly the best motivation to keep doing it with the same intensity) so it’s great to hear it from other people too. And it gets even worse – once we collapse, there aren’t going to be the concentrated ores and high EROEI fossil fuels to power the development of another highly technically developed civilization after that; we will have used them up. It will take hundreds of millions of years for such resources to build up again. So it’s one and done for us and maybe for the planet. People talk (nowhere nearly as much as they should, of course) about our sustainability crisis but they much rarely discuss its deep implications and it has to change because we lose perspective of things this way.

    One thing you have not discussed (maybe you haven’t done that just yet but will in the future) is population. I see no way to power down without some draconian population control measures. We can talk about living in sustainable way all we want, but if there are 10 billion of us, there is simply no way for us to be sustainable. Obviously, that’s a very touchy subject but there is no possibility for ever achieving sustainability without addressing it.

    • The “sustainable” buzzword may make Western professionals and academics feel good about themselves, but it isn’t much consolation to those actually living the resource-free “sustainable” lifestyles championed at cocktail parties and faculty lunches in the West.

      There’s a reason why standards of living drive so much of our world’s conflict. Owning a personal vehicle and living in your own home helps make life not be a burden. By contrast, living in a communal straw hut and borrowing the village ox to run your errands is always going to make for a crappy existence, no matter how much we might champion it as being “sustainable.”

      Western nations are up in arms that the birth rate is, in many cases, below the replacement rate. What is the problem?

      Western nations also turn a blind eye to mass immigration from third world nations, which serves to ease resource pressures there; in effect, is is an export of surplus population in a manner which increases consumption.

      What we need to figure out is how many enjoyable, comfortable lives can be supported, then determine how to get there. Short of global totalitarianism, it will likely require aggressive immigration controls for the non-educated, otherwise any gains in sustainable reproduction in the first world will be more than wiped out by haphazard reproduction elsewhere.

      The personal comfort genie is out of the bottle. It can’t be put back through social policy, no matter how hard some may try. Our only option for a conflict-free future is through figuring out how to work with the standards of living already demanded, which will require fewer people on this planet.

      Stopping government subsidization of childbearing is a logical first step.

      • Without wanting to enter into the “class politics” mentioned above… I think a big problem is that many people don’t quite get that being unsustainable means it must all come crashing down — unsustainable _means_ collapse is inevitable.
        It’s kind of like saying “well, yeah, this can’t keep going forever” without realising we’re being painted into a corner.

        • [shortened by moderator]

          It would be great if the masses viewed sustainability in a mathematical sense. I used the scare quotes for a reason. I’m tired of seeing “sustainable” abused as a euphemism for greenwashing some silly consumer product or as a euphemism for a poor but low impact lifestyle.

          I’m not trying to dredge up issues of class ideology for political purposes. The natural world has its own way of quickly shrugging us off when we take to the podium and blow out politically charged hot air. The planet will give us either many, hard lives or many fewer good lives. The planet can’t support ten billion of us living as Western consumers.

          To me, sustainability is a system which isn’t necessarily destined to crash. A system which requires the developed world to devolve to the world average, or even those near or below average remain frozen in their opportunity, is destined to crash, because of fundamental tenets of human nature.

          I get frustrated when I see so many well-meaning people think we’ll all go peaceably towards riding bikes, shutting off the climate control in our homes, and eating vegan diets in harmony with our fellow humans. The real world doesn’t work like that.

          Power down is ugly, very ugly. I’m fully energy self-sufficient, I don’t intend to power down, and I hope my neighbors don’t either, for then I become the target.

          • Don’t confuse discussions of technical feasibility with naive belief that it will be popular. I’m one of those people recommending bicycling, carpooling, eating less meat, and turning down the thermostat. That encompasses a whole lot of powering down, right there, and the reason I bring it up is that it is all technology that we could deploy on any given morning, the investments required are incredibly low, and it is also stuff that we can try out to see how we like it. That we’re not all already doing this supports your point — people resist change.

            On the other hand, when energy gets more scarce, we’re going to get price signals, and those are probably the first things that people will do — because the investments are low, and because people are cheap. The fact that they need no lead time also means that they help reduce the risk of being “trapped” without the energy to get out of a hole — at least to the extent that those choices will save energy.

            And I would like to get people “softened up” to the possibility of doing these things. Cutting consumption early slows the onset of scarcity; that gives us more time to plan and prepare (to do all the other effective conservation steps that DO require investment and lead time).

        • Well, there’s transition to a renewable modern high tech economy; there’s collapse; and there’s soft degradation to a lower but historically still high tech economy. Regression to the 1950s via paying more for electricity is not the same as collapse to the 1750s and not having electricity, but it’s hardly clear that the latter is more likely.

          IIRC a study said that people in Manhattan use 1/3 the energy of the US average, between the transit efficiency of subways and the thermal efficiencies of apartment buildings. Italy also uses 1/3, though it’s both poorer and benefits from a more clement climate; still, it’s a First World country.

    • There’s that question: can 10 billion people live sustainably on the planet? Well, I think the best answer is, “that depends”. It is well known that as standards of living improve and societies become more structured, birth rates go down (to below replacement often). So the question becomes, will we be able to raise people’s real standards of living (not in terms of absolute consumption, but in terms of social fairness, justice, and economic opportunities) in time and in a sustainable way before the system collapses. Because once it collapses then socially we we will go back in time — unemployment and poverty will go up, and then we might see a resumption of high birth rates.

      Let’s say hypothetically we were able to get a solar energy infrastructure developed before things collapse. Then the whole of North Africa and the Middle East could become a giant energy powerhouse, they could use this to desalinate sea water and then irrigate their currently unproductive deserts. Then global food production could probably double and be able to provide a comfortable and sustainable standard of living to everyone.

      But if society and economies fall apart before this infrastructure is implemented, will we be able to afterwards? We have (had) an opportunity to use our remaining fossil fuels to do so, but I fear that because of social, political and economic dysfunction, it will not be pursued and we will be doomed to fall off the cliff, especially since as mentioned, once scientific literacy and knowledge is lost, then the chances of us building out such a renewable infrastructure become even less.

    • That made me think of a Frank Herbert quote:
      “Beyond a critical point within a finite space, freedom diminishes as numbers increase. This is as true of humans as it is of gas molecules in a sealed flask. The human question is not how many can possibly survive within the system, but what kind of existence is possible for those who so survive.
      Pardot Kynes in ‘Appendix I: The Ecology of Dune'”

      Of course in our case, the finite space is a contracting one…

  4. Tom,

    Great blog. You’ve presented some great ideas about energy production, but what I’m most interested in is our transportation future. Any thoughts on that, or plans for a blog post to cover that? Considering a very large portion of our current energy outlay is spent on transportation, any idea of what a post-fossil fuels transportation network could entail?

    • Reading between the lines of this post, I think it is safe to say the answer will be “Walking”.

      • No, bicycling is more efficient than walking, whether carrying yourself, or carrying a fair load of cargo.

    • I think this recent recession was a little taste of economic collapse. I for one am glad that I moved to a downtown location in ’05. I am able to walk, bike or drive my car to close locations. I have also seen the increase of people using bicycles to get around, even in winter and bad weather.

      Point being, that I think the suburban sprawl of post WW2 is going to stop and recede back to an urban setting. Long distance travel might be something you do only once every 10 years or so. Look at what life was like in the 1920’s and apply that to now with some modernization. I see people squeezing into smaller cars and not driving SUVs for 1 or 2 people. More telecommuting and working from home.

      • telecommuting is an interesting subject, I’m still baffled that that hasn’t taken off. 25% of all jobs could probably be done from home these days…

    • US uses 3 TW of energy. Roughly 1/3 of that is fossil fuels burned in relatively inefficient mobile heat engines, generating maybe 300 GW of work in actually pushing cars around. If you have something like solar generating electricity directly for an electrified transport system, you need to replace the 300 GW we use, not the 1 TW we burn. And the electrified transport system might well need less power, if using mass transit, slower speeds, trains, regenerative braking, etc.

      I’d expect the post-fuels network, if at a high end, to look more like Japan or Europe than the US, but even more so.

      Worth noting that with gas taxes of 100+%, such countries have been living in an expensive-gasoline regime already, so they’re adapted, and have room to cope with rising prices by unwinding gas taxes (though the government will have to raise taxes elsewhere to make up for the lost revenue.)

    • Provided we develop an ample supply of high temperature fission reactors, we can efficiently synthesize the liquid fuels (energy carriers) we need. At a global scale, it would appear that the most suitable fuel would be moderately toxic ammonia (NH3). We can also synthesize carbon fuels (methane, methanol, dimethyl ether, synthetic gasoline, etc.), which may be affordable and carbon neutral with adequate biomass feedstock. A direct-ammonia fuel cell car is a type of electric car with comparable performance to conventional vehicles.

  5. I will be working in moral, political and legal philosophy for at least the next decade to argue for a renewed conception of the moral legitimacy of the constitutional order in democratic-liberal polities which is acknowledges the environmental crisis and its threat to civilization.

    The main objective of the argument, and then of the constitutional reform, is to court-circuit the “political suicide” problem. We have an extraordinary complex and difficult collective action problem, both at national and global levels. But it must be overcome to achieve steady-state economy within the limits set by the Planetary Boundaries theory (Rockstrom et al., 2009).

    Redesigning political and legal institutions for the XXIst century is in my opinion one of the best tools we have for self-behaving,

    • John Michael Greer, at the ArchDruid Report ( http://thearchdruidreport.blogspot.com ), has the most cogent argument that I’ve seen about practical change. He practices—of all things—symbolic magic, which he believes is not about influencing physical reality (the realm of science and engineering) but rather that of influencing masses of human beings. He maintains that our society is dominated by black magicians who manipulate the masses through things like advertising, “dog whistle” politics, appeals to base instincts, religious brainwashing, etc. He maintains that we are doomed to suffer a stepped, slow motion collapse.

      He says that it is impossible to manipulate people for the good, because the method of manipulation inevitably changes and debases the manipulators. (I read in the Guardian today about a scientist who used dirty tricks to leak documents about the Heartlands dirty tricks campaign to discredit global warming: http://www.guardian.co.uk/environment/2012/feb/21/peter-gleick-admits-leaked-heartland-institute-documents?intcmp=122 . )

      I disagree somewhat. I think that what might do some good is exactly the sort of work that this blog is doing. I despair of the fact that so many decisions in our society are done by people who haven’t “done the math”. My hope is that the internet might be able to raise the awareness of enough people that they might be able to do some prep work to avoid a total collapse. This is Greer’s hope too, as he is trying to encourage people to learn “hands on” skills that will allow them to thrive as the economy slows down, contracts and the larger society thrashes around like a wounded buffalo. (See: http://www.greenwizards.org .)

      • Unfortunately, I think Greer suffers from a different flavor of the same apocalyptic thinking that he so concisely diagnosed in many other flavors in one of his articles.

        We stand and fall together. The only way out is with all the others, not through or ahead of them. John Brunner once wrote that every human being lives in a cage, the bars of which are all the other people. Libertarian thinking assures us others do not matter. Localitarian thinking – looking forward to the comeback of yesteryear in a long and comfortable emergency – assures as of the same, on a tribal scale. The world has gotten smaller than that.

        By all means, prepare. But recognize that it will amount to nothing unless its amounts to an example for others, helping them realize that change is needed. Democracy, flawed as it is, and incompatible with compound wealth as the current version has proven, is the only way out, because there are no benevolent dictators, only oligarchs, and the defining characteristic of inbred wealth is its inherited stupidity.

        • I agree. Living a sustainable lifestyle by yourself might feel good, and getting your local community involved can feel even better. But it’s not going to save the planet, and if the larger culture really starts to break down (from lack of liquid fuel or water or climate change or whatever) they’ll just destroy whatever “unspoiled” patches are left.

          I tend to agree most with Derrick Jensen (http://www.endgamethebook.org/Excerpts/1-Premises.htm). He argues that it’s not enough to simply offer a sustainable alternative- we need to actively fight against the paradigm of consumption-based industrial civilization and seek to destroy it.

          • Whilst acting alone, or with local community, will not save the planet, it is something each of us can do. I think it’s a cop out to argue that because the planet won’t be saved (whatever that means) by individual action, we should wait until there are enough acting together to make a difference. If no-one acts, that will never happen.

            Derek Jensen asks where are you going to place your picket pin; well we each have the ability to place it somewhere, so we should.

        • I am glad to see Greer’s blog mentioned here. That and this forum are both highly valuable, and to some extent complementary. The more people who read both, the better.

          Nothing about Greer’s writing should be exempt from criticism, but I would be careful how you phrase it. You are in danger of glibness and imprecision.

  6. The most pressing problem is indeed the coming liquid fuel shortage. Non-nuclear electricity alternatives are not yet at (widespread) grid parity today, but that is purely a cost issue. Even though the ideal technology isn’t there now, people will begin implementing less than perfect technologies at higher than desired costs when the alternative is a power down.

    Government has tried to pay lip service to the problem, but all we get are impractical boondoggles like the Volt and the Tesla, which only serve to squander resources which could have been used for more practical research and construction of more practical products.

    The best approach for handling the coming liquid fuels shortage is through an aggressive implementation of plug-in hybrid electric vehicles. This is currently the best technology which allows electricity to substitute for liquid fuels.

    Why isn’t adoption faster? We’ve run into a chicken and an egg problem. Greater uptake can’t happen without public certainty in affordable electricity rates (contrary to Obama’s campaign promises), but without that uptake, the incentive for developing better and more affordable PHEVs is diminished.

    One glimmer of hope has been the rollout of TOU metering. This will be crucial in making the substitution of electricity for liquid fuels cost affordable.

    With the PHEV currently the only proven technology which allows us to translate electrical abundance into an extension of the usable life of liquid reserves, we have to aggressively embrace this technology.

    The only way that will happen free of boondoggles, malinvestment, and crony capitalism is a certainty in affordable electricity rates. Hence, nuclear.

    Government can spur certainty in the payback offered from grid charging of vehicles by getting its boot off the throat of the nuclear industry by aggressively issuing combined licenses for designs proven safer than our mostly-1960’s nuclear stock, while simultaneously assuring a minimum amount of bureaucracy for those developing advanced Gen-IV designs or thorium fuel cycles.

    Nuclear is the kickstarter. There will be time for renewables when they reach grid parity, our grid becomes smarter, and we can handle the intermittency. The cause of those renewables would be greatly served by having a fast-developing fleet of PHEV customers already in the pipeline when the issues with renewables are solved or lessened.

    Nuclear is all we have capable of breaking the chicken and egg deadlock.

    • I am curious why you argue in favour of PHEV’s yet you characterize the Volt and Tesla as boondoggles? Isn’t the Volt a PHEV? While it may not be everyone’s cup of tea, it’s a pretty fine car. And the Tesla, however impractical it may be (no more so than the Smart Car, Mini Cooper or Mazda Miata — I see quite a few of those “impractical” cars out there) forever destroyed the myth that EV’s are wimpmobiles. With Nissan’s Leaf (I own one), every single possible objection to the impracticality of EV’s has been shattered, except for the range limitation issue, which can be easily addressed. I’d say the EV and PHEV movement is coming along nicely. 3 years ago I would not have expected such progress.

      • [shortened by moderator]

        The Volt and Tesla are subsidized toys for rich people. Both are pure electric and only got built because they made for good press for politicians. Even a PHEV Prius is cost-prohibitive for normal people.

        What we need to avert the liquid fuels crunch is a PHEV family vehicle with a reasonable payback period. For that to happen:

        1. The extra cost must be reasonable, like an A/C option or a sport leather package. This criterion will likely only be satisfied in cars which are already built as hybrids.
        2. Electricity rates must be certain. We can’t have politicians promising to make electricity rates “necessarily skyrocket.” Even if you believe such promises are good policy, they are heavily counterproductive in the short term when it comes to getting ordinary middle class people to power their VMT with grid electricity instead of liquid hydrocarbons.
        3. Battery technology (the chemistry) must advance from where it is now, which won’t happen as long as priority is given to building proof of concept sports cars for hedge fund managers, partially at the government’s expense.

        The Leaf is a very expensive car for what you get in intrinsic value. I’ve test driven the Tesla, the Leaf, and obviously the Prius, and in the end I decided my money would work hardest putting more solar panels on my home and upping my insulation. Find a way to add 30 miles of grid-powered all-electric range with a three year breakeven point and we’ll be on our way to replacing many liquid fueled VMT.

        • I’ll agree that the Volt is an expensive toy — I test drove one at a car show recently, it’s definitely a luxury vehicle — but you’re incorrect in stating that it’s pure electric. It is a hybrid; check the Wikipedia page.

        • I disagree that the Volt and Tesla are subsidized toys for the rich. Firstly, the entire GM operation is subsidized by the gov’t so it’s not fair to single out the few EV’s it produces. Secondly, the point of sale rebates arguably even out the systemic subsidies that gasoline enjoys which artificially bring its price to $4 a gallon, or whatever it is in the US right now.

          And the fact that rich people are the first to adopt new technology is hardly news — that’s how all new things come to market. Then mass production and innovation reduce costs so that average people can buy into it.

          Regarding the Leaf price, considering where gasoline prices will likely be going over the lifetime of my car, I think my initial capital investment today may pay off to be more than break even over the long term, in comparison with a $15,000 regular car.

          I think battery chemistry is fine right now, especially for PHEV’s with gasoline backup.

    • The best technology is probably mass transit, which uses proven technologies at largely acceptable costs, not things substantially more expensive than the cars they’re replacing.

      My approach would be to raise gas (fossil fuel in general) taxes steeply, on a fee-and-dividend system so people have the money to pay the high prices, but do face the high prices to incentivize adapting, combined with big investment in public transit — light rail, electric buses — and maybe encouraging more ridesharing programs, or cheaper (subsidized?) electric taxis.

      • Mass transit is a non-starter until the crime problem is dealt with.  If people are assaulted on the train or bus or thieves use it to spread to new territory, nobody will want it to come near their town.

        • [edited the fangs out]

          [Whoa, there!] Are you suggesting that everyone who currently uses public transit is either a criminal or so “beyond the pale” that they don’t care about all the thugs they ride to work with every day?

          I had a half-dozen acquaintances die from auto accidents when I was in secondary school. This is far from an isolated experience for people who live in the country. I haven’t known anyone killed in a robbery. Why is it our society chooses to “swallow camels while choking on gnats”?

          One of the great things about this blog is that the author really does try to “do the math” instead of making moronic statements based on emotion. 🙂

      • For energy savings buses are worse than cars (same drive train), even single passenger cars. For full cars the bus is not even close. All those frequent stop/starts, making the rounds on light passenger days, and inevitable return empty to the bus barn add up. One still saves energy by jumping on an *existing* bus to help fill it (over a car), but the purchase of that *next* (marginal) bus, not so much. Congestion reduction is another matter.

          • Energy used (BTUs) per passenger per mile:
            ————————————————————
            Cars: 3538 (decreasing at 0.8% per year)
            Personal Trucks: 3663
            Motorcycles: 2460
            Buses: 4242 (*increasing* at 1.4% per year)
            Rail, commuter: 2812
            http://cta.ornl.gov/data/chapter2.shtml
            Table 2.12

            I see McKay has his bus point at 32 kWh / 100p-km, and single passenger cars at 80 kWh / 100p-km. The US bus transportation figure (4242 btu / p-mile in the same terms is 77 kWh / 100p-km, US cars are 64 kWh / 100p-km. It may be that the data for US cars uses *average* passenger load.

          • Ah, turns out US car data is for a passenger load factor 1.55 persons per car, and buses with a disappointing 9.2. Likely the population density difference of 8:1 and other factors fill up the buses a more in the UK, yielding better energy efficiency per p-km.

          • Not just density (and 8:1 is misleading; what matters is the density local to where people live, which is still lower in the US but not as much; the US having lots of empty land is irrelevant) but the design of the system. Most US buses aren’t even designed to be a primary transportation system, but are a sop to poor and old people. Don’t have a car? There’ll be a bus eventually, if you’re willing to wait. Naturally only people who have to take them. It’s transportation welfare, of course it’s subsidized.

            UK is designed more for cars being a luxury.

            So yeah, just doubling the US bus ridership to 18 — compared to a capacity of 40-60 would put it well under your cars figure, or even the commuter rail.

          • @Falstaff – you really ought to add to your list:

            Upright bicycle, commuting pace (~11mph): 200 (source: “Bicycling Science”)

            Electric scooter, same pace: 60-80 (estimated)

            Electric scooter, actual pace (22-33mph): 120-400 (further estimated)

            Estimate reasoning: Humans are about 25% efficient converting food to mechanical energy, plus they idle at 300 BTUs/hour (so 27 BTU/mile is “overhead” at 11mph ). This gives 44 BTU of actual work. A bicycle at commuting pace sees roughly equal parts wind and rolling resistance (22 BTUs each). Scooters have much more efficient electric motors (80%? including battery charging), but also tend to have less efficient wheels, so ding them for that, because bicycle rolling resistance is about half the drag at a commuter pace. Wind drag work is quadratic in speed for a fixed distance, so it quickly dominates. Double speed, 22 BTUs becomes 88, triple speed, 22 becomes 198. (I also sanity checked the scooter high-end against known human power output and speed records — it takes 350-400 watts to propel a bicycle at 30-ish mph).

            The cost to produce and deliver the energy to the motor varies wildly; scooters run on coal-electricity are still a better GHG bet than cars run on oil because they are so small. The energy cost of producing human fuel varies by a factor of around 100 for common sources, from 85% beef (assuming you consume the fat, 30mpg-effective, or 4000 BTU/passenger-mile) to oats (assuming cooked on a wood stove or solar cooker, 3000 mpg-effective, or 40 BTU/passenger-mile — at those levels of efficiency, how you cook the food matters). (Source: USDA calorie estimates, “Food, Energy, and Society”)

            It would seem, if our goal was to save as much energy as possible with unicorn-free technology, that “electric 2-wheelers” (scooters, and assisted bicycles) are worth a really hard look. 400 watts is hard to beat. They’re popular in China http://dr2chase.wordpress.com/2011/01/03/e-bikes-in-china/ and apparently increasingly popular in the Netherlands (so I read). Because the batteries are so much smaller, they are not limited by wall socket capacity (i.e., 110 vs 220 is not an issue), but more by the battery’s own charging rate. In some cases (not all) batteries can easily be swapped, which gives another option.

            Interestingly, I just returned from a brunch get-together of local family/cargo bikers, and in the discussion of “why”, not one person mentioned peak oil or the environment. Most of them do it to save time, to save money, because it is convenient, or because it makes them feel good (i.e., better health). This would suggest that attempting to guilt people out of their cars is not likely to work that well; addressing the “why are these people not preferring scooters” problem is likely to work better.

        • Depends on the bus. Near where I live is a “trackless trolley” (electric bus) that exists in a virtuous cycle; people move near the route because of the bus, and the bus runs often because of all the people who use it. It’s routinely half full or better even in the middle of the day, and at rush hour (when it runs every 5-8 minutes) it consistently fills up. These busses are old and lack regenerative braking, but it’s an obvious thing to do that would make them even more efficient.

          • What’s the point of the Bogota link? Buses can be unpleasantly crowded, yes. So what? Maximal efficiency can lead to unpleasant crowding, as anyone who’s flown knows. Is this supposed to be an argument against buses? It’s not like you need such packing for buses to be more efficient than cars. Having maybe 1/3 of the seats filled suffices.

          • re: the Bogota link. You’re perhaps right, that may be irrelevant. The busses I ride (Boston) are never that unpleasantly full (usually not even close).

            On the other hand, I’ll bet that is the perception that many non-bus users in this country have, and the fact that actual busses got to that state is not so good. It reinforces the idea that busses are for people who are poor or who have no choice. You could not ask for better utilization; what level would be required, to get more busses run to alleviate the crowding?

            It’s also not adequate to be merely more efficient than today’s cars. It would be nice to aim for 3x more efficient, or more.

          • I live in Cambridge; depends on the bus. The frequent ones are more likely to fill up; the trains even more so.

            Efficiency? Look at the discussion with Falstaff:
            Cars: 3538 (decreasing at 0.8% per year)
            Personal Trucks: 3663
            Motorcycles: 2460
            Buses: 4242 (*increasing* at 1.4% per year)
            Rail, commuter: 2812
            But that was with mean bus ridership of 9. 18, which still doesn’t fill even half the seats, would be 2121, far more efficient than cars as used (at 1.5 ridership) and more efficient than even motorcycles. Filling the seats would have us at 1000 or below. Crush capacity, well!

            Of course, part of the efficiency is from not moving fast, not just having lots of people.

          • I’ve been more lax than usual on keeping the discussion focused on the post topic. This one on bus/train transportation has run its course, I feel, and will be dropping further comments. I should probably do a post specific to transportation efficiency at some point.

    • Greater uptake can’t happen without public certainty in affordable electricity rates

      I don’t agree with this at all. With the Leaf getting 3miles/kWh, the cost per 100 miles, at 10c/kWh is $3.30. With a similar sized normal car getting 30mpg, at $4/gal, the cost per 100miles is $13.30. You could *quadruple* electricity rates and still be cheaper.

      People are uncertain about EV’s because they have shorter range, are expensive, and “unproven”. The cost of electricity to operate one is trivial.
      This sounds like you are using it as a straw man to argue for nuclear, but let’s be honest, if achieving this rate certainty you require for EV’s means placing a nuke plant *upwind* of every major city, how much support do you think there will be for EV’s?

      We can power a very large fleet of EV’s – charged in off peak hours – with the electrical infrastructure we have today. In fact, just doing so will make electricity cheaper, on average, because of higher capacity factors on generating stations and transmission lines.
      Adding more baseload nukes makes the peak/off peak disparity worse, not better.

  7. Safety of all is not assured, but it appears to me that the safest exit has a vocal contingent blocking it, claiming it is least viable because most passengers would agree with them. This is neither a trivial nor technical problem.

  8. “I have to have a “spidey-sense” of what things are likely to succeed or fail—…”

    Tom,
    I laughed when I read this. In my line of work, a keen “spidey-sense” is a necessary attribute – I laughed because I’ve never heard anyone outside my profession refer to it.

    Every day when I go to work, I have to be ready to solve real-time problems that can be measured in terms of volume (scale) and complexity. Even in low volume scenarios, complexity alone can make for challenging work. When you combine high complexity with high volume, the “operation” can quickly get out of control – entropy will run wild. Therefore, at some point, decisive action must be taken to simplify the “operation” prior to “the wheels coming off” – this is where the “spidey senses” should kick in. The brain needs to be subconciously “ranging out” into the future and “doing the math” so that you get that feeling that it’s time to rein things in BEFORE it all blows up in your face. The “solution” in the most general sense invariably incorporates both putting a cap on volume but also actively reducing complexity by restriction. Since the imperative of the “operation” is public safety, sitting on your hands and assuming that it will all work itself out in the end is obviously NOT an option.

    What worries me when I contemplate all the “big picture” stuff is that my “spidey senses” are screaming at me, yet I see no decisive action (appropriate to the scale and complexity of our civilization’s predicament) being taken that will have the required effect in a timely fashion. It seems that despite the implications for public safety, there is no collective will to do anything except let it all play out on its own… Yikes.

    I sympathise with you when it comes to relating your concerns in public but I am glad that there are some such eminently mature people such as yourself attempting to ring the alarm. What’s so bad about being an “alarmist” anyway?

    Lastly, here is an animated short that I recently came across that I think provides a pretty good summary of the important issues related to energy/growth – though the title is a little unfortunate in my opinion:
    http://www.youtube.com/watch?v=VOMWzjrRiBg

    Cheers.

    • That video is very depressing, unfortunately true. I like the advice in the last minute or so, as I see that the be the real future. Now the timing of this collapse is hard to pin down, could be 2050 or 2100. But it will happen and life will be a lot simpler when it does.

    • After re-reading my comment, it occured to me that I should have probably defined the “operation” as air traffic control.

      Something I was thinking about earlier today, further to my earlier comment, was that the best way to “solve” an extremely complex problem in real time is not to search your toolbox and debate what is the best tool to use but to first attempt to stop the problem from evolving to greater magnitudes and higher levels of complexity.

      If the problem can be stabilized in this way THEN it makes sense to make a nuts and bolts evaluation – selecting the appropriate tools in an attempt to “solve” the problem.

      I won’t hold my breath waiting for this problem to be “solved” by committee…

      In the “Dune” series, Frank Herbert’s character Leto II accepts mutation into a wormlike being of great power that lives for thousands of years. He does this because his oracular powers allowed him to see that without the ability to plot a particular course through thousands of years of the future, humanity would be doomed to self destruction from shortsightedness – not being able to foresee all the concequences of their decision making.

      I wonder if the world we’ve made for ourselves isn’t already so complex that to “solve” it requires some kind of god-like foresightedness combined with the ability to take decisive action.

      I won’t hold my breath for this either. Personally, I think we’re all going to end up flotsam caught in the currents of stormy sea.

    • I can only agree. As an engineer I have found out many times that the devil is in the details, and many solutions that look good on paper show their warts only when you start playing with them, or sometimes even after they shipped and are in the hands of customers (Fukushima?) You get a “spidey-sense” of things that look too good to be true.
      Why does Murphy’s Law come to my mind? Hee hee.
      We need to try all of the above and see what works. Yes it’ll cost a boatload of money. The alternatives are worse.

      • Ralf,
        I think you’re right.
        The devil is in the details.

  9. This is an excellent and well balanced summary of the situation we are facing. I agree especially that substantive change can come only from the bottom up. Politicians are too heavily invested in the old order and there is little chance they will promote policies that will hurt the financial interests of the elite. I believe the political class is incapable of leading change of this magnitude. To quote Ghandi: we must become the change we want to see in the world.

    This is precisely the kind of conversation all of us should be having across North America and around the world, from kitchen tables, around water coolers, in boardrooms, community associations to town-council chambers and beyond.

  10. What are the minimal-plus-a-tad-of-quality requirements for life? Rev. Thomas Malthus himself opined it should include a piece of beef, a glass of claret etc. every once in a while. Besides shelter and clothing to keep the cold out and decency within, along with food, there are other needs. My Dad regularly told folks about how, in the South Pacific in the early 1940’s (not a vacation paradise at the time), after food and letters from home, what the young Americans wanted most was entertainment. Dad liked to say that we need to not only make a living, we also need to make living worth while.

    What is on the other side of the «exit doors» in your analogy? By this, I mean energy and resources that we have assuredly available now or in reasonable time? For instance: if we decide food is more important than flying to where the young Americans were but were not vacationing in the early 1940’s, we should have enough petrostuff for agriculture for a while… but how long a while?

    There is some thinking to be done before the math can be done. Otherwise it’s the one about the nonswimming mathematician who drowned while wading across a little river with an average depth of eight inches. Maybe it’s time for a game my kids used to play: “What-if, Let’s say…” − and scope out future possibilities.

  11. I was talking to a friend about population and remarked that we needed a half percent per year decline starting as soon as possible, and they said “oh, we need more than that when the Hubbert curve starts to head down”.

    I said I hadn’t picked that number because I thought it was nice, I picked it because I thought it was the best we could do voluntarily, assuming eighty year lifespans and one child. Imagine a “Children of Men” crossed with mild “Logan’s Run” scenario, where we have no children at all, and also contrive to force people to the grave around their late sixties: how fast does the population decline then? An impressive one and a half percent a year. And yet Tom thinks we might see energy availability fall faster even than that.

    • That’s some scary math. I hadn’t considered before what a percentage decline in population would look like in real human terms.

      If fossil fuels decline 3% per year and we have to cut our population 3% to match, that would mean basically that everyone dies in their 40s with no kids.

      • Things are bad enough without inventing new troubles 3% decline in fossil fuel can be met by reducing the per capita usage, not just by reducing the number of capita. And the former is a lot more likely than the latter.

        • [edited]

          Humans aren’t going to go quietly into their sustainably tiny studio apartments deep within city limits and close to the bus stations.

          Most of them will cling to their wishful hopes of abundance until the crisis is at hand, at which point society will devolve into a competition for scarcer resources.

          Never understimate human greed or malice. Know this: in all of human history, nothing has proved as powerful at stopping these negative behaviors than sufficient economic prosperity to afford homes, cars, meat, vacations, etc.

          When people are content, most choose to play by the rules.

          People become more dangerous as they have less to lose.

          Take away the resource abundance, and we’re left with 60 hour workweeks followed by bad food, bad TV, [etc.] and not much else. Do you want to live in a neighborhood of people living like that?

          • “Humans aren’t going to go quietly into their sustainably tiny studio apartments deep within city limits and close to the bus stations… Most of them will cling to their wishful hopes of abundance”

            Ah, modern urban apartments of whatever size and good transit (bus stations? I don’t think you know what urban mass transit is like) IS abundance for the vast majority of the human race. What would be a step down for a middle class American would be a step up for most… including many Americans.

            “sufficient economic prosperity to afford homes, cars, meat, vacations, etc.”

            We passed that level of prosperity quite some time ago. It’s no longer being equitably distributed, though. And supposedly wealthy Americans have crappy vacation time, and some would say crappy but cheap meat too.

    • (Showing my work here)

      If the world population has to decline 3% a year, and people completely stop giving birth, that would mean 3% of people die every year, 97% survive. A 97% survival rate means you have only a 0.97^40 = 30% chance of reaching age 40. Or perhaps the survival rate would be higher for young people, but then it would have to plummet after age 40 (Soylent Green anyone?)

      If people can’t stop having babies, and we keep giving birth at 2% a year, then people would have to die at FIVE percent a year to reach 3% population decrease. At 95% survival, you’d have only a 0.95^20 = 36% chance of even making it to age 20!

      • Or we could, y’know, just use less. It’s not _that_ hard for many Americans to cut energy their use by 90%. Much, much easier than no kids and suicide at 40!

        • The whole point of this blog is that it’s NOT that easy to “just use less”. Certainly not 90% less- and we’re talking about the whole world, not just America. Right now if America burns 3% less oil I’m sure the rest of the world would be happy to burn 3% more. A peaceful, long-term, worldwide reduction in fossil fuel use would be such a massive undertaking that it would dwarf WW2.

          • At the risk of putting words into Tom’s mouth, that has not been the whole point of the blog, to date. It has been that it is not easy to meet our current energy consumption through non-fossil sources.
            He hasn’t – to date- really focussed on how to reduce usage, though he is effectively saying we will have to, its just whether we jump or are pushed…

          • “The whole point of this blog is that it’s NOT that easy to “just use less”.”

            I think you’re wrong. I think the whole point of this blog is to show that business as usual (eg. current levels of energy consumption) cannot continue without fossil fuels, and that lifestyle/expectation adjustments are required.

            Take my word for it — it’s easy to use less, but it does require planning. We have a 1.5 kW solar system (very small) and it produces more electricity than we use. We do drive, about 2400 mi/year, and the rest of the time cycle. We eat (fairly local) organic food (ie. few petrochemicals required). We don’t buy a lot of stuff.
            The point of this is not to boast, but to show that a few minor lifestyle changes have a big impact on energy consumption. We do not live a life of poverty by any measure, and can yet do much more to reduce energy use.

          • Soon enough, I will be advocating a “use less” approach. I’ve done it myself, and see the (reduced?) power in it.

          • I did botecs a while back on a high quality nation-wide bus system. I’ve yet to get anyone to read it enough to give a thumbs up or down on the assumptions and math.

            http://mindstalk.livejournal.com/220864.html

            I don’t spell it out there, but the moderate mass transit everywhere scenario is still using less gas than the current auto fleet, and the buses could be electrified via pantographs or whatnot. Or replaced by more sensible but harder for me to estimate light rail.

      • It need not be a constant rate decrease. Pretty much everyone alive now will be dead by 20100. If we want the population to decrease down to, say, 500 million or less before the year 2100, we just make sure not much more than 500 million are born and survive into adulthood between now and then. The rest will die naturally, most of them in the latter part of the period, but this is not a problem because if you manage to persuade people to agree to such a scheme, persuading them to consume less per capita will be a trivial matter. Of course, this means no retirement, most people going childless, etc, etc. Needless to say, it’s not happening, period. The population crash to below 500 million will most likely happen though, the question is how ugly it’s going to get, whether will manage to avoid going down all the way to zero, and if yes, in what condition will the survivors be…

  12. Thank you for your hard work and great series of posts. Thank you for taking the time and words needed to fully express your feelings and ideas.

    For those who haven’t seen it yet, I would like to recommend Low Tech Magazine at http://www.lowtechmagazine.com/

  13. The state of modern political conservatism dismays me. Disconnected from reality, disconnected from the numbers and unable to use the power of numbers to do more with less. My wife and I are old school conservatives. We routinely live within 1/3 of our income. Yes it is possible. The rest of our money we save or give away. We don’t need it. That is the nature of conservativism, not to use or do things you don’t need. When did Greed become a conservative value. And to ‘take’ the Earth, gifted to us by God, and turn it into an exploited wasteland become virtue.

    Meanwhile I would point out that much of the world operates in intermittent power. In fact the modern western world with 24/7 uninterrupted power is very much an exceptional situation. Less than 25% of the world’s population enjoys access to this uninterrupted unlimited power. Collapse need not mean a destruction of the world, merely a return to how the remaining 75% of the planet lives. IIRC correctly the average Indian/Chinese 20 years ago lived on 1/30th the resources the west uses. So that collapse situation very much exists on the Planet right now. If the supply of oil/energy supply drops to 1/30th the present level, that is how the western world too will have to learn to live. Even at 1/30th the present energy level, energy use is many orders of magnitude higher than the energy levels available even 300 years ago when horse and manpower alone drove everything.

    If renewable’s can supply even 30%-50% of present energy in an intermittent manner as seems likely, it would put a permanent back stop to the collapse scenario from an energy point of view.

  14. I’ve advocated nuclear here (and even space, long-term), but I completely agree. We’re in a race to transform our energy supplies, and we might not win. And yes, the more we conserve, the better our chances.

    What’s terribly frustrating is that so many people who are concerned about these things (not you) dismiss one of the few viable solutions. “Nuclear? We don’t need it. Renewables will do just fine.” Well, what if they don’t? I’m all for rolling out renewables as fast as we can. I’m just not convinced they’re enough, and I don’t want to see nuclear left off the table. Abundant nuclear isn’t ready yet, but that’s all the more reason to get started.

    Regarding transportation: in the short term, replacing fossil power plants would free up more fossil fuels for liquids. In a pinch we could synthesize liquids from coal or natural gas, using process heat from nuclear. (For climate reasons I really hope we avoid that.) Longer term, all the options are pretty speculative at this point, but include better batteries (zinc-air, lithium-air), synthetic fuels from atmospheric CO2 (nice because you keep the same fleet and infrastructure, but difficult since CO2 is sparse), ammonia (easier to make from air, but serious safety issues), and boron combustion (more energy dense than fossil fuels, recycles with process heat, but needs new engines). For much more efficient transportation, personal rapid transit looks interesting:
    http://climatecolab.org/web/guest/plans/-/plans/contestId/5/planId/15207

    I think we should develop all of this, as vigorously as we can. I’d also be very interested in your take on these guys, promoting a very efficient and large-scale biofuels scheme at Google’s Solve For X:
    http://www.youtube.com/watch?feature=player_embedded&v=zkYVlZ9v_0o

    • What’s terribly frustrating is that so many people who are concerned about these things (not you) dismiss one of the few viable solutions. “Nuclear? We don’t need it. Renewables will do just fine.” Well, what if they don’t?

      My sediments exactly (“if you aren’t a part of the solution, you’re part of the precipitate”).  So many “environmentalists” still toe the old “peace” party line, anti-nuke across the board while both James Lovelock and Patrick Moore have seen the light.  Nuclear power has been cutting carbon emissions for 50 years, and would have done far more if not for their ignorant opposition.

      Regarding transportation: in the short term, replacing fossil power plants would free up more fossil fuels for liquids. In a pinch we could synthesize liquids from coal or natural gas, using process heat from nuclear.

      Nuclear heat can also make the processing of biomass easier.  Subcritical hydrolysis can break lignocellulose down into separate streams of 5-carbon sugars, 6-carbon sugars and lignin; the first two can be fermented, and lignin has industrial uses.  Low-grade heat can distill fermentation products to fuel.  The fossil fuel freed up is another bonus.

    • Great comment. I’d just point out that CO2 is not sparse when it comes from limestone kilning in Portland cement manufacture, and the current world production is on the same order as what you’d need for synfuels to replace air and long-haul transport. The Gen-IV nuclear plant next door to the limestone kiln supplies the process heat for kilning, high temperature electro/thermolysis for H2 (maybe S-I cycle) and the subsequent steps to generate whichever liquid is most economic/convenient (MeOH, DME, etc.). Get the tech and price straightened out on U238/Pu239 or Th232/U233 (or maybe fusion one day) and the chem-eng and scale up for carbon-neutral liquid fuels should get taken care of at a reasonable price point and EROEI.

      Biofuels: walk away, unless it’s a currently polluting waste stream you can recycle, which is a small resource. Coopting inefficient photosynthsis for transport liquids is a bad idea. We need the land and sea for biodiversity and its services, and for food.

      • >>>Biofuels: walk away, unless it’s a currently polluting waste stream you can recycle, which is a small resource. Coopting inefficient photosynthsis for transport liquids is a bad idea. We need the land and sea for biodiversity and its services, and for food.>>>

        Walk away from corn ethanol perhaps, though I would not walk away from more efficiently grown liquid fuels (algae, cyno bacteria, etc) should they become practical using non-arable land and non-potable water. After all corn ethanol currently uses 60,000 sq miles in the US and its hardly missed. Cover that same land with 500 bbl/acre-year crops and … well, do the math.

        • Algal / cyanobacterial biofuels aren’t ready. Even if they are one day, they’re likely to be temperamental, as well as requiring phosphorus, nitrogen and other nutrients. When it comes to splitting water, if you do the math, I think you’ll find that the efficiency of C3 photosynthesis is sufficiently low (a few %) that the same non-arable land would be more productive if covered with CSP. On a large scale, for simple molecules (such as fuel) with cheap feedstocks, chemical synthesis invariably works out to be a better deal than a biochemical alternative.

    • So, personal rapid transit looks interesting, but it would be also interesting to compare its capacity and performance to some of the alternatives (i.e., there’s some math to consider). So, how long is a PRT vehicle? What is their following distance? What’s the capacity of a loaded Acela (or Megabus, what have you) from Boston to New York? (Says Wikipedia, 8800 people per day ride the Acela Express, not sure how they count that). If PRT pods travel 30 feet apart (and you might want them to draft, for efficiency), that’s 50 miles of PRT (distributed in some fashion along the Acela route, which extends all the way to Washington DC).

      Locally, there’s the issue of getting to/from the PRT, versus speed of PRT. Best case from my home to MIT by car is 18 minutes (Google predicted it, I timed it early one Saturday morning) not counting time to park. Bicycling, or bicycling to the red line and walk to my destination, is 30 minutes. If I walk 1/4 mile to a station at 3mph, that’s 5 minutes, plus the 12 minute ride (6 miles at 30mph) to get to MIT, or 17 minutes, plus walking at the MIT end. A local bus has 3000 inbound weekday boarding, running into Harvard Square, with the heaviest use in the morning rush. At a 30 foot headway, that’s 17 miles of pods in a line. Does that work?

      Alewife (Red line endpoint) has 10000 entries — again, weighted to the morning rush — or 57 miles of pods at 30 foot spacing.

      (A great source of transit data — the MBTA Blue book: http://www.mbta.com/uploadedfiles/documents/Bluebook%202010.pdf . I’m using 2009 figures, since it’s a bit of a download. )

      Another rule of thumb is 1500 cars can pass over a spot on the road, per hour (so says an ex-traffic engineer friend; this rule can be violated, but traffic engineers get interested when it is. He remarked on this because there was an intersection in Houston where a patch of pavement achieved 1700 cars/hour).

      I’m sure I am missing some assumptions about provisioning and distribution, but it doesn’t look like it’s necessarily an easy win, compared to some of the alternatives that are already deployed.

      I should add, that for cities, I recommend adding bicycles to the mix. Maybe not for everyone, but for many people (anyone thinner/fitter/younger than the fat 50+ year-old that is me, how about that?). Cheap, easily deployed, proven in other countries, fast enough for <= 6 mile urban-ish trips (compared to measured speeds for cars and transit), flexible, compact, efficient. And we need the exercise — especially if we are fat and/or 50+ years old.

      And I would also ask of PRT, the same question that is asked of anyone recommending bicycles — how would I carry my groceries on that? (On a bicycle, you play the "but I ride a cargo bike" card, which is good for 6+ bags. What's the story for PRT?)

      • Only 1500? i’d seen 1800 — 2 second following distance, basically. And with rush hour carpooling, maybe 2000 people. That’s freeway though; city streets probably halve that, because of cross-streets.

        Cargo bike is one answer; a different one is “cargo trailer”. More flexible since you can take if off, though I don’t know how it behaves going downhill…

        Buses seem to tend to be 40-60 people. I think I estimate 40 seats for MBTA buses, which run small. Greyhound buses are 50 or 55 on their website. Then there’s standing room though I’d be happy to pay extra for a system that was “inefficiently” comfortable. US headways tend to be long, but elsewhere in the world one can see 2 minutes or even 90 seconds. Boston’s hitting that with the Red Line replacement buses. PRT can apparently have fractions of a second in theory, say 5400 vehicles an hour — or half the capacity of a light rail line.

        • No need to estimate — that blue book reference has THAT data, too — the more common trackless trolley has 31 seats, crush capacity of 68, planning capacity of 43. The most common (and oldest) bus in the T’s fleet carries 40/77/56.

          I’ve tried trailers, I’ve tried cargo bikes. I like cargo bikes better. It has to do with your attitude both towards planning and towards riding a bike; if you need a trailer to carry stuff, you’ll only carry stuff when you make a plan to carry stuff. Cargo bike handling depends on the cargo bike; if you insist on the snappiest, crispest feel, you won’t like a cargo bike, but a longtail is otherwise a comfortable, stable ride. A trailer is okay, till you tump it over (done that) or snag it on an obstacle (done that — only a risk when you are towing, but risk is higher when you are since it is not normal case). A trailer can, however, allow you to haul a load that would be impractical even on the most capable cargo bikes (e.g., a full-size ladder); in a bike-full world, I’d assume that either the big-stuff stores would stock a few, or there’d be someone in your neighborhood who owned one (or you would have big stuff delivered).

          Point I’m trying to make is that there are no one-size-fits-all solutions, but also that I have tried several (I’ve tried the bus to work outside of 128, bleah). Bikes are extremely good urban transportation by most technical metrics, but in this country, not very popular.

  15. This is a fantastic summary Tom, I now see why I agree with your blog more than with any other. One comment I have though concerns your statement that you support balancing budgets. In theory I like this idea, but the problem I have is that our current monetary system which is driving this insanity is fundamentally flawed because it requires perpetual exponential 3% growth to function, and this is the only way budgets can be balanced on the long term. To try to balance the budget is to service a flawed system.

    While I am quick to point out how the field of economics suffers from a systemic misunderstanding of the basic principles of how the world works, I think this also goes the other way to some extent and affects scientists and engineers, in their general misunderstanding of how the monetary and economic systems work. I’ve been twisting my mind over the last few years trying to bring economics and science together and I think I’ve basically done it, but in doing so I find myself pulling my hair out over all the misconceptions I see out there.

    As an example, here in western Canada, there is a huge drive to export our oil sands to China. Of a total deposit of 1.8 trillion barrels, around 200 to 300 billion are recoverable with EROEI greater than one. Divide this by 30 b barrels a year global consumption, and you get that the oil sands represent 6-10 years of consumption … from the largest petroleum deposit in the world! I point this out to fellow oil sand-gung ho engineers at work and I get the glazed look and standard response, “Well, with today’s technology maybe. In the future we’ll be able to overcome this”.

    So we have a whole office of professional engineers seemingly incapable of picking up a calculator and doing basic math, and having faith in our ability to violate the laws of thermodynamics. I find this even more inexcusable than the economists’ fantasies, because they simply weren’t taught the knowledge to be able to know better, but engineers were. The human mind is a strange thing.

    This is a good video I recently saw regarding how our brains are currently malfunctioning:
    http://www.youtube.com/watch?v=dFs9WO2B8uI&feature=player_embedded

    • Regarding economics versus the real economy: In our modern, interlinked monetary system *all* money is debt, at a positive interest rate. If there is not exponential growth in the real economy at something close to the prevailing interest rate then within a short period of time the bankers own everything. Shortly after that they tend to lose their heads.

      Money, as a means of exchange and store of value, is very useful even if a positive interest rate cannot be sustained, but cannot be based on debt at interest. A steady state, or shrinking real economy requires a complete reinvention of the monetary system.

      • [shortened by moderator]

        I think this essay from 1999 is relevant here; it certainly enlightened me about the nature and effects of the current money system, and possible alternatives:

        http://www.feasta.org/documents/moneyecology/intro.htm

        Here is an excerpt from that intro:

        This Briefing will show that history is littered with examples of monetary systems that operated on quite different lines to the one we know at present.

        And another excerpt:

        Chapter Four (One Country: Four Currencies) attempts to weave all these threads together by devising a multi-level multi-currency system which would ease the world’s transition to sustainability by improving the way which the economy allocates scarce resources between the present and future generations. It proposes an international unit-of-account currency whose value would be based on the right to emit greenhouse gases.

        ===

        I [the commenter] think “the right to emit greenhouse gases” is in principle too narrow — if this makes sense, I’d guess it should be generalized to something like “the right to cause serious future problems for humanity (of any kind)”, which in political practice would mean (if ideally democratic) “the right to do things which lots of other people think will cause serious future problems for humanity”. But limiting it this way might be better than nothing and a decent way to get started.

    • “While I am quick to point out how the field of economics suffers from a systemic ”

      I have never met anyone in the energy decline movement who has even a basic grasp of the elementary elements of economics. This includes the “ecological economists”. Energy decline adherents sometimes claim that they are familiar with the field of economics, but I can tell right away that they are not, because they do not correctly repeat even the basic elements of the field. Maybe you are the exception; maybe you’re an expert about it. However I’m very skeptical of claims about that field from energy decline forums.

      “our current monetary system … is fundamentally flawed because it requires perpetual exponential 3% growth to function”

      Our monetary system does not require any growth to function. Some countries (like Japan) have had levels of growth near 0% for decades without collapse.

      “professional engineers seemingly incapable of picking up a calculator and doing basic math, and having faith in our ability to violate the laws of thermodynamics”

      The energy decline movement has repeatedly claimed that collapse is imminent, a consequence of arithmetic and simple laws of physics and thermodynamics. If this is true, I must ask why they have had a 100% failure rate of prediction. For example, Charles Hall (who wrote the foundation of much of modern energy decline theory) wrongly claimed in the 1970s that EROEI was declining and it was impossible as a matter of thermodynamics to increase net energy. Then, net energy increased by a factor of 10, over the next several decades. What happened to thermodynamics?

      The problem here is that none of the claims from the energy decline movement are really derived from thermodynamics. Instead, those conclusions are derived from thermodynamics PLUS a series of unstated economic and logical assumptions which are common-sensical but wrong.

      “Of a total deposit of 1.8 trillion barrels, around 200 to 300 billion are recoverable with EROEI greater than one. Divide this by 30 b barrels a year global consumption, and you get that the oil sands represent 6-10 years”

      Is it necessary to have an EROEI of tar sands greater than 1? Does thermodynamics require this? Only if tar sands are the only source of energy. Otherwise we could use breeder reactors to process the tar sands into oil. We could do this even with an EROEI for tar sands of (say) 0.8, since the _combination_ of breeder reactors and tar sands would still have an EROEI of more than 20 in that case.

      Only if the _aggregate_ EROEI from all energy sources falls below 1 does it imply energy collapse as a matter of thermodynamics. That’s not inevitable, because we have available enormous and practically unending sources of energy (like solar thermal) with EROEIs higher than 10.

      Maybe the economy will fail to transition to other (more plentiful) sources of energy as fossil fuels gradually decline. Perhaps the economy will just collapse instead. However this is an economic calculation problem, the answer to which cannot be derived from thermodynamics or simple math.

      • Certainly thermodynamic arguments are invoked more often than they are relevant. Seldom does one see an actual calculation involving entropy using actual units of J/K. For what it’s worth, I’m comfortable with the original DtM post that absolute energy growth on Earth leads to a thermo-death, but this is a pretty extreme set of assumptions (that we would grow at 3% for another few centuries).

        And it is also certainly true that EROEI < 1:1 does not mean impossible to extract, as long as other EROEI > 1:1 sources are available for the I (input) part.

        So from a physics point of view, I can’t prove we will collapse any more than I can prove that we won’t. The battle is on sociological, societal, and yes, economic grounds (which is just a reflection of the previous two).

  16. [Moderator note: this comment is way longer than I generally accept, b. But I’ll give you the one-time experimental physicist discount and let it through this time (also true that I don’t have time to edit down presently).]

    Decades ago I used to be an experimental physicist. Tom Murphy’s text is an excellent expression of my thoughts on these issues as well, so maybe it is the training, and the experience of building finicky devices, that informs this kind of conservatism. Most people consider me a radical.

    I would add a few footnotes.

    For one, the irrationality – to the point of self-destruction – of human beings, individually and collectively, is exemplified not just with respect to climate change, but even more visibly and immediately with respect to so-called economics. It has been glaringly obvious since 2008 that the dominant economical models – ultimately based on the assumption of rational actors with total information – were, unlike the harmonic oscillator, violating the principle that you should simplify as much as you can, but not more. It has also been obvious that systemic fraud on all levels – including that of so-called economic science – will corrupt democratic institutions and ultimately destroy any economy. Even if there was no connection between high EROI and easily accessible transportation fuel on the input side, and economic growth on the output side, a dysfunctional parasitic debt economy is not sustainable. Economists might not want to believe that, but anthropologists appear to have the record on their side, at least according to Graeber.

    Take analysis like these:
    http://www.hussman.net/wmc/wmc120220.htm
    Week after week after week his comments are basically attempting to justify a conservative approach to dysfunctional and corrupt markets. This is an issue that directly targets the wallets, and should resonate with any modicum of common sense. Yet, most of us are indeed out there skating on the thinning ice.

    For another, I continue to believe that one of the most important aspect of any sustainable energy supply is a discussion of the corresponding economical powers structures – both those needed to create the infrastructure, and those that will result from it. This is an important issue for nuclear power, but also for other potential parts of a solution. I have mentioned before the work of Alexander Rossnagel, who during the German 70’s debate regarding pervasive use of fast plutonium breeder reactors considered the implications of weapon-grade and high radiation fuel cycles – and the resulting security apparatus – for an open society. There might be a lot of unwarranted hysteria regarding nuclear fission for power generation – and many warranted concerns regarding fissionables in the hands of corporate profiteers – but there is a lot more, and a lot less warranted, hysteria regarding nuclear terrorism, a hysteria that has become part of the unsustainable and suffocating growth of the National Security State. An RTG in every village has very different implications from gigawatt-scale nuclear plants.

    The same concern – the economics of power generation, distribution, access – is true for large-scale solar infrastructure. Who owns it? Who controls it? Is our future a dozen Enrons before the blackout? Distributed vs. centralized power generation is extremely important, not just with respect to societal acceptance, but also with respect to sustainability of an open society. All other aspects equal, it is unlikely that our society would choose solutions that “empowered” the individual, because the political structures that “govern” our collective decision making have been distorted by undemocratic incentive structures, to the point that we not only no longer work towards the common good, but are abandoning and dismantling our industrial infrastructure for the personal paper profits of a percent of a percent.

    In a way I am faulting Tom for debating what is, by his own admission, a psychological, sociological, *human* problem, in technocratic terms. It does not matter how many doors are left without goats behind them if we cannot do the math, and cannot bring ourselves to walk through them. As a corollary, I also consider most of the “local watt” thinking as escapism – libertarian solipsism, really. The world at large will run out of oil, heat up, and otherwise collapse no matter how many of those that are paying attention are able to reduce their use of energy to negawatts in the splendid isolation of their homes.

    In Vernor Vinge’s “Marooned in Realtime” – the book which introduced his extrapolation of Moore’s Law before it became Kurzweil’s rapture of the nerds – there is a metaphor presented to illustrate that, for a technology-driven civilization like ours, there is no such thing as a temporary setback on a global scale. It is based on the image of a scramjet engine stalling at high altitude, running out of atmospheric oxygen, and how the pilot has to negotiate altitude into velocity on the way down to re-light it before augering into the solid ground. Our growth-based economy is the most obvious expression of this risk. This is not tai chi – we are not balanced at every moment of our upward movement, and concerns about science – or hundreds of years of infrastructure, and dozens of generations of education, craft and training – should be enough by themselves to silence those that assure themselves that human ingenuity and unregulated markets will always save the day.

    As long as The People cannot find the motivation and a way to effectively stand up for the rule of law, and a functional economy – growth ranks a distant third to these foundations, if at all – there is no prospect of climate change or peak fossoil being addressed before we run out of means to match our ends. On the other hand, 2008 is not over yet, and maybe once we are forced to face that, our collective decision making will mature to measure up to the long-term issues as well. If nothing else, the generation that will have to clean up our mess will have no sympathy for our generation’s glibertarian approach to survival.

    In my father’s house, there are many rooms. Surely, there must be one without an elephant?

    • You correctly note that this is a sociological problem.

      I am currently an undergrad in Sociology, and until recently I have been content with the idea of going into academia. However, after watching “Collapse” and later stumbling upon Prof. Murphy’s blog, I am finding it much more difficult to see rationality in pursuing a career that very well may not exist by the time I’m looking for tenure.

      On the other hand, my family owns ~300 acres in central Kansas. And while it’s culturally desolate and politically hostile, I’m beginning to think that my long-term survival would be better served by self-education in solar technologies and organic farming.

      I love the academy and being around a diverse group of critical thinkers (something my hometown clearly lacks), but again it comes down to the question of what seems more rational given that society is already beginning to lurch and stumble.

      Any thoughts you, other readers, or Prof. Murphy might have as to this dilemma are greatly welcomed/appreciated.

      -InTheLurch

      • Boy, I wish I had access to 300 acres in Kansas…

        I have myself contemplated whether to stay the academic course, as disconnected as that world can be from here-and-now problems. But I do think academics could play a key role. We have and support a talent pool in this country partly to tackle the big problems when they come along. Clearly physicists, chemists, etc. can contribute on the technology end. But sociologists, psychologists, and the rare ecological economist could end up playing even more significant roles in helping to flesh out how a society can cope with a transition away from fossil fuels, which may involve dealing with less and hopefully will result in a steady-state economic framework.

        It’s a wide open field: no one knows how it will play out, but academics could shine a flashlight into the dark and expose some possibilities that might otherwise go unnoticed.

      • Reasons to study sociology include figuring out what causes the crowd to make choices that are apparently self-destructive, and how to fix that. People do all sorts of weird irrational stuff, both individually and collectively (my wife’s more or less in the social sciences, I get to learn about stuff like the Lake Wobegon Effect and the Fundamental Attribution Error). It’s also darn amusing that we’ve been measured to have “dependent utility functions” (keeping up with the Joneses — if Jones gets a raise and I don’t, that makes me unhappy. Why?). Wouldn’t hurt to bone up on your Robert Frank (“Choosing the Right Pond”, “Winner-take-all economy”, “Luxury Fever”) to get a feel for how this relates to economics. (The fact that we have dependent utility functions, AND irrationally optimistic estimates of our future, AND loss/gain asymmetry, means that the field of economics has some pretty shaky mathematical foundations. “If you can divide by zero, you can prove anything” — and no surprise, they do!)

        OH. Read also, “Normal Accidents”, by Charles Perrow.
        And add to that, “The Control of Nature” by John McPhee.

        Those acres in Kansas, I’d evaluate them from the POV of do they have water, do they have wind, do they have sun, and what’s predicted for their likely climate in our CO2-enhanced future?

      • Jon, I hope you don’t make major life decisions based upon presumptions of energy collapse. If the collapse doesn’t happen then you will have cheated yourself out of a lot.

        Why not just keep your 300 acres as a backup. You would have enough time to relocate to Kansas later, should the need arise, because the collapse almost certainly would be very gradual. I don’t think anyone is talking about sudden collapse anymore, since peak oil appears to have occurred 8+ years ago without collapse.

    • “the image of a scramjet engine stalling at high altitude, running out of atmospheric oxygen, and how the pilot has to negotiate altitude into velocity on the way down to re-light it before augering into the solid ground. Our growth-based economy is the most obvious expression of this risk.”

      Claims that our economy must either grow or collapse are common in energy decline forums. Although I’ve never seen the scramjet analogy before, the basic claim has been made repeatedly elsewhere.

      The claim is incorrect. The economy does not require any growth to avoid collapse, and in fact could endure fairly severe _contraction_ without risk of collapse. For example, the US economy contracted by 25% from 1929-1933 and did not recover until after WW2, 15 years later. Yet it did not collapse–it remained an industrialized country throughout. Similarly, Japan has had severe economic problems for decades now and has not grown by any appreciable amount for 20 years, yet it has not collapsed, not even after a massive earthquake during which it lost much of its power generation and distribution capability.

  17. I agree with Bill.

    Mental exercise: say you want high-speed rail between two of your choice cities. Imagine, in your mind, two scenarios about building it: One, best case, that it’s a matter of upgrading some tracks and smoothing out some curves. Scenario two, worst case, is that every foot of track has to painstakingly travel through established high-income neighbourhoods, impossible gradients, and across pristine wetlands full of endangered species.

    High speed rail is difficult enough that Scenario 2, for the non-technical person, might as well be true in all cases. Especially the ones that seem simple on first pass. I am a civil engineer involved in transit systems planning btw.

    The author keeps warning about pie-in-the-sky thinking. Living with less energy may mean longer travel times while we take the slow electric streetcar to work. But again and again, we will have to live with less, not more of anything. We need to allocate what few billions we have into light rail and sustainable electricity.

    • It has been done. We have seen the past, and it worked. And of course we could always do it better.

      In the days of streetcars and choo-choo trains, we got relativity, we saw the universe expanding, and listened to jazz. In the days of sailing ships and animal-powered drayage, we crossed the globe, charted continents, learned about logarithms, calculus and more, and produced great art.

      And we can throw the past away. We’ve done that. We can also throw futures away; done that, too. But we have more options than Bang or Whimper. If we choose to choose.

  18. A collapse scenario is quite possible, but it also seems that once the problem is undeniable that a lot of change will be made that will allow a form of civilization to survive. During WWII when Japan and Germany started losing fuel resources from 1942 onward from direct lose of the territory and increasingly effective bombing of German resources and sinking of Japanese shipping but they lasted to 1945 when they were physically occupied or atomic bombed. Germany even developed jets and rockets during that period. Of course it has been argued that the Japan and Germany would have collapsed in another year without being occupied, but then they were taking a shock in a year what would probably be 10 years under a slower loss of resources.

    • Both Japan and Germany employed extensive slave labour (The V2 killed more people in its construction than in its use as a weapon of war) and it is widely considered that German ‘wonder weapons’ were a colossal waste of money. Freeman Dyson thought the V2 was so expensive next to conventional, propeller powered, bombers that he considered its development to be an act of unilateral disarmament. The Nazi’s arsenal of rocket-powered suicide planes didn’t do them much good either.

      If anything, the collapse of the Nazi military despite their development of some undoubtedly clever technology is a warning that last minute technological fixes might simply be futile.

    • I have read that eventually Germany’s fuel crisis got so bad that the Luftwaffe issued orders that the jet fighters were not to taxi out to the runway for takeoff — they were to be towed by horses.

        • Fascinating reading. Maybe for a future blog Tom could look into the experience of Imperial Japan as a precedent for a liquid fuels crisis?

  19. Thanks for writing such a clear, accessible, and informative blog. Unfortunately it only feeds my general pessimism with numeric backing, but I’ll let my kids worry about the long-term stuff…

  20. The way is shut when the many are oblivious.
    The way is open to the “1 percent” in control.
    The answer is in a focused mass awareness.
    The solution is unlimited energy at full throttle.

    We MUST figure and agree on the BEST possible plan which will be complicated, confusing and very arguable to most.
    An energy source with the best EROEI, the least toxic (per joule), and the most abundant seems the obvious choice.
    Electrified mini rail or cable may be the best for transport of goods, yet wouldn’t it be easier to develop personal rapid transit (or carpods on cable) for all of us, and continue to use diesel powered big rigs in the mean time to prevent collapse?
    This approach bypasses the “necessity” to attain a battery, hydrogen or other energy intensive liquid fuels infrastructure (to a large degree).
    This also addresses a lure for 3-d cities, the backbone to future prosperity (and fun, imagine the built in roller coasters and the vertical modes of transportation)!
    However, advanced nuclear (such as LFTR) does not guarantee the economic growth (also) necessary to avert partial collapse. This is why we need to build 3-d cities and the robotics needed to help us manage massive hydroponic based food supplies within.
    What better way to ease the transition into an obviously fast approaching machine economy (assuming collapse not withstanding)?

    The tech is present, just not the coordination…

    • “This is why we need to build 3-d cities and the robotics needed to help us manage massive hydroponic based food supplies within.”

      I cannot imagine anything worse than living in that city and eating that food.
      Our current industrial food system is bad enough, and you want to make it employ even less people and produce even more artificial food?

      • You probably have a good point about not wanting to live in a highrise (no matter how architecturally beautiful and no matter how much energy it would save by ridding suburban transport). I would rather live on acreage (but can’t afford to nevertheless).
        If people lived there, they would want pesticide free food and what better efficient way than hydroponics?
        I guess we wouldn’t need machines to help with gardening, but I said that because I assume that “everything” will involve machine automation by then anyways.
        Perhaps the machines would really be needed to “3-d print” the actual buildings (from some exotic material like carbon graphene even)?
        I understand that some kind of Luddite law is necessary, but I don’t understand how it will work. Even today, burgers can be flipped by machines, they just aren’t because of bad image (thankfully). Kiosks are already at Southwest gas and Jackinthebox. Eventually, “all menial jobs” will be lost to machines, so what do we do?
        I used to think that we HAD to use machines to make solar panels just for this very reason… to create (sooo many) more install jobs. But, it seems we do not have the time and energy to do “all that” before the oil crash.
        It’s looking like the plane has to crash twice so to speak! So, not only do we have figure and agree on the “best” energy and transport plan, we also have to figure out how to successfully enter into a machine economy where massive jobs loss is anticipated.

        • “Eventually, “all menial jobs” will be lost to machines, so what do we do?”

          Decrease the work week and tax the wealthy. If technology is supposed to displace labour (which is touted as a good thing), then the only way to keep employment up (without economic growth) is to spread the remaining work out more.

          In our current capitalist system, achieving grotesquely inflated states of personal wealth on the order of magnitude equivalent to many entire countries is seen as an admirable goal and something to be strived for. However, this is dysfunctional and was not typical for most of our history. Tax rates for the super-wealthy used to be up in the 80-90% range, but Reagan reduced them in the 80’s and they have basically gone down ever since.

          Since the laws of physics stipulate that wealth is not created, but rather taken and transformed, from the (finite) natural world, it therefore follows that those who have net worths of billions of dollars have somehow, in one way or another, taken that wealth away from the rest of the population (ie, via runaway wealth concentration a la the board game Monopoly).

          Of course, the way this extreme wealth concentration is justified to the masses whom are being robbed is that if we were to increase tax rates for the super-wealthy then no one would have any incentive to work …. OK, so when everyone becomes billionaires then they will have no incentive to become multi-billionaires because it just gets taxed away. Let me know when everyone becomes a billionaire and is frustrated about paying 90% tax on their next billion.

          Everyone’s being thrown into poverty right now.

  21. In emergent systems such as society and economics, the myriad interactions that contribute to the system can be beyond effective consideration in predicting the final outcome. Attempts to educate the masses are yet one more set of variables that further confuse the picture.

    Top-down solutions run into problems of unconsidered interactions that can affect the outcome either way. The other option is to encourage bottom-up emergent behaviors in the hope that the system as a whole has tendencies in the desired direction.

    Since the current political and social climate is quite adverse to the desired top-down methods, the option to pursue is to nurture the desired bottom-up efforts.

    Some of these considerations are discussed in the EconTalks podcast interview with Nassim Nicholas Taleb on Antifragility:

    Taleb on Antifragility

    The issues of preserving infromation in the digihal age are presented in the this talk by Clay Shirky at the Long Now Foundation:

    Clay Shirky, Making Digital Durable – SALT

    • “Attempts to educate the masses are yet one more set of variables that further confuse the picture.”

      I think much more pressing is the full-scale campaign to delude the masses into thinking that every thing is A-OK.

  22. Tom – I wonder how you respond to the techno-utopians of the Kurzweil variety. Regarding energy, his claim (and that of others like him) is that solar PV is advancing exponentially in terms of Watt/$ (and consequently in deployment) so by the mid 2020s we’ll be using 100% solar PV globally. (And he claims that advancement of such technology, like Moore’s Law, isn’t affected by recessions.)

    Of course he ignores the many infrastructure issues, substitutability, and all the others that David Fridley sums up nicely here: http://www.postcarbon.org/report/127153-energy-nine-challenges-of-alternative-energy

    While I don’t buy it I suppose Kurzweil could be right; that would make my concerns about fossil fuels, and those you’ve described, seem silly in retrospect. (I’m interested in such predictions because I like to identify blindspots in my thinking, but I’ve had difficulty in reasoning about why or how he might be right or wrong.)

    • I hope he’s right, but on the face of it this sounds like ruthless extrapolation ignoring many practical concerns. I would have to look in more detail to get a sense of the “reality factor.”

    • “He claims that advancement of such technology, like Moore’s Law, isn’t affected by recessions.”

      In first order approximation, Moore’s Law has been about economy – it measures how much investment into silicon manufacturing advances we can afford. It also measure how much money we have to continue to replace perfectly workable computers.
      Worse, it has already changed. In the words of Herb Sutter, the free lunch is over. It is no longer possible to exploit increased density by increasing clock cycles, as dealing with the waste heat is no longer.. economical. Hence, we are forced to branch into concurrency, and there are good and sound reasons why concurrent programming was not preferable to increased clock cycles. Amdahl’s Law limits what we can do with the silicon, even if the dynamic Moore observed continues to deliver more silicon at prices that, at the least, do not increase as much.

      Ultimately, physics places limits on Moore’s Law as well. You can’t split the atom here.

      Kurzweil is no utopian, he is a cornucopian – a space cadet of a different kind. If somebody handed us the blueprints for a fusion reactor that would take a million times global GDP to build, all the technology and science in the world would not make it happen. In fact, we already have such a blueprint, and a working model in the sky. To claim that technology on its own – without the energy and labor to implement it – is sufficient is just another flavor of Chu’s technium/hopium/techopium.

      We cannot turn ingenuity, innovation, science into power – unless we burn e.g. the Astrophysical Journal. But then National Geographics alone might get us through this century…

  23. Some stats for Japan’s population between 1944 and 1945 show a drop from 73.9 to 72.0 million (http://www.populstat.info/Asia/japanc.htm) or from 74.4 to 72.1 million (http://en.wikipedia.org/wiki/File:Population_of_Japan_since_1872.svg). I’m not sure whether to believe those numbers, because that would be one of the great famines of history, but I can’t find a reference to any such thing. It’s a population fall of over 2.5% in a single year.

    It’s possible these are just wrong numbers, and the downward blip and bounce back are bad counting in one year corrected in the next. I can well believe the bounce back of over 4.8% in 1946 is real, though. I’ve said that populations fall very slowly unless people are dying horribly, but the same isn’t true of population rise. 5% only requires one woman in ten to have a baby that year. Famines, war, and disease are sudden catastrophic events that can kill tens of millions in a year, hundreds of millions in a decade, but show barely as a blip in the statistics, because the missing people can be replaced within a decade at the most.

    For that reason I expect that without a conscious effort at family planning, a sort of Kyoto Protocol for baby emissions, the world population will always bump up against the ceiling of food supply, with very brief periods of global abundance in the immediate aftermath of a catastrophe.

  24. I really like this post tying it all together. You’ve convinced me. What remains is a road map for where to go from here. When we begin hitting oil shocks, it is going to be much easier to convince people to begin building safety rafts. The problem is that just as many people that will be convinced to pump the last few drops of oil even faster.

    That’s why having a plan is essential. People are going to want to do something…or have something done. With a plan we can maximize our chances of them doing the right thing instead of the wrong thing.

  25. You just voiced my thoughts. I am also physisist (Ph.D.) and also work in designing complex systems, and I regularly run through the same arguments. I even think the same way about Science. I got my Ph.D. in France but moved back to Russia, as according to emergy analysis Russia will be hit the least of all countries in the coming crisis.

    As I see it now, humanity is going into collapse for sure. Though the situation will be very different in different parts of the globe. As for my country I am pretty sure that we will conserve a kind of civilization through coming century. Will we manage to move to nuclear after our 50 years+ gas resources would perish? – I don’t know.
    ( No solar or wind would help when it is -30C outside.)

    At least I have some hope, that during the collapse of many societies abroad, people here will collectively do something so that we don’t follow their path. And just to illustrate this point I vividly remember a conversation with an executive from RosAtom (Govermental Nuclear Corporation) when she said: ” Though it is not economical at present we will continue developing breeder reactors. “

    • Alexey,

      It is scant hope, but “humanity is going into collapse for sure” is an assessment of the nature of human beings. There is no determinism here with respect to the numbers: We have the resources and the opportunity to prevent or mitigate collapse decisively. What is lacking is our collective ability to get our $#!+ together.

      That is an important distinction for me. The certainty that human stupidity might govern our future is different from the certainty that it is already too late based on the numbers. In that gap lives whatever hope is left. Or, in other words: People matter. It’s a curse, but it’s also a blessing.

      • As I point out most often to annoy cliched RPG, fantasy, and SF writers, much of civilization is Stone Age technology, based on moving dirt around in the right ways. Sewers, roads, irrigation canals, urban water supplies, granaries for getting through bad years, basic record keeping, (textile) suspension bridges… you can have unwashed barbarians in the early Steel Age (medieval), chopping each other up chivalrously and piously, and you can have vast egalitarian bureaucratic civilizations with all of the above amenities at the dawn of stone or early Bronze civilization (Harappa, Andes… okay, not so egalitarian for the Andes.)

        It’s all about the social organization, which can be a thought for optimism (we can do it!) or pessimism (odds are obviously poor.)

      • I’ve almost no hope on preemptive action of humanity to mitigate the collapse. Surely, we will react on it, but it would be too late. It looks like the situation in 1920, wise people expected a new big war in europe in 10 years +, but couldn’t do much about it.

        I cannot even agree with you about: ” We have the resources and the opportunity to prevent or mitigate collapse decisively.”, there is no “we”, no governing structure that could see a path to follow.

        Though I know one person in history who did a similar task: Stalin in Russia put to poverty and into jails a great part of the population, but was able to industrialise the country in 20 – 30 years. He sold abroad bread, making russian people starve to death, but bought whole factories abroad during the Great Recession. As I see it the task of mitigating the collapse imn advance would be similar to this.

        I can’t see a power on the global scale, that would be able to do this.

  26. I follow this blog, but it seems to me that several things aren’t covered in enough detail, or at all.

    1) Long distance transmission of electricity. My perception (feel free to disabuse me) is that HVDC is good enough to get the job done. It would be nice to have superconducting cables, but HVDC is good enough. Literally this is a game changer. There is a lot of wind in coastal Alaska. With HVDC it is in range of the Pacific Northwest and Western Canada.

    Of course these places have significant wind resources of their own.

    The same thing applies with wind on the Great Plains, and Chicago. Then we can do the North African Med solar farms and Europe. (they can jolly well do their aluminum smelting and high energy whatnot in the daytime only)

    Just throwing out ideas, but why wouldn’t they work? I have more, but the point is HVDC is a game changer that doesn’t seem to be discussed enough to me.

    2) As a physicist do you think it is impossible to make batteries that won’t meet the requirements we need to fill the surface and even water transport needs we have?

    We aren’t necessarily close, but a breakthrough in this field has enormous ramifications. I have no idea how research money is allocated, but if this were a game of Alpha Centauri you can bet I’d be prioritizing this field.

    Just saying it wouldn’t take much (opinion) to really shake things up.

    • I believe we might have to replace the bulk of the “fossil fuel burners” with electrified mini rail and ditch the battery altogether (and keep the big rigs on diesel) at least for the time being. I was hoping there was a better solution than (im)personal rapid transit, though.
      Maybe there are enough battery raw materials, afterall, the global lithium resource needed for the LiFePO4 battery seems to just barely meet all global transport needs… This battery has 2,000 complete charge/discharge cycles, can be charged in 1 hour and discharged even faster. Although more energy dense, ordinary li-ion can not even come close.
      I assume if humanity wanted, machines could make it for pennies on the dollar.

  27. Thanks Tom, always enjoy your writing.
    About the way out, and what each and everyone can do, which I expect will be (one of) your next post(s): I tend to agree with the conclusions from “sustainability without the hot air”. We all need to reduce our energy use in a way that is significant on our personal scale. In short: Unplugging the phone charger at night -> pointless
    but
    plastering your roof with solar cells, biking to work and vacationing at the nearest shore instead of indonesia -> significant difference

    keep up the good work!

    • Felix, I second that.
      One of the things I like about Tom;s blog is the no-nonsense way he approaches things, which is the same as David McKay. And the conclusions, not surprisingly, are similar – niche things, no matter how “good” they are, cannot prevent the coming energy crunch.

      Unfortunately, the niche things get lots of PR, and are being used to lull many into a false sense of security.

  28. Great post!

    I miss the exergy in this blog: posting such a topic would be great.

  29. Thanks for creating “Do the Math”!
    Your work (and the comments) galvanizes me to “do something”.
    I know that conservation is good, as it helps to develop efficiency… But we can’t live on conservation alone.

    Perhaps “cars” on electric cables, hydroponics, etc, powered by load leveling advanced nuclear, will become the exits.
    We might NOT have to build 3-4 times renewable energy capacity and the need to build its storage (I never thought I would come to this conclusion because I “love” solar and wind energy).
    With personal rapid transit, we can also eliminate the need to create all out battery systems for transportation, too.
    The electrification of the “global fleet” greatly extends the remaining (easy) fossil fuels for big rigs, industrial activities, etc. until advanced battery systems become commonplace.
    Currently, these doors are closed because they are hidden behind the “first class”. We need to collectively figure the best plan and broadcast it before the smoke gets too thick…

  30. I agree that it is VERY difficult to explain peak oil or the transition to a post-carbon future in a short 1-2 minute “elevator pitch” to friends and acquaintances. I’ve tried various approaches (trying to sound matter-of-factly optimistic), but most people start looking at me like I’m seeing ghosts after about 30 seconds.

    I would have thought it would be easier to convince people if you are a physicist/professor – it’s really sad that people don’t even want to hear it from you!

    I’m planning to print out business cards with a few selected peak oil websites (including this blog) to just hand to people instead of trying to explain it.

  31. Environmental considerations are still being missed here, though some posters have mentioned environment. The energy predicament is part of a wider predicament that encompasses many fields. Without a response to those other factors, it really doesn’t matter whether there is a satisfactory response to the energy predicament. We live in the real world and there will always be a consequence of anything we do. When deciding on the mix of energy sources (though I think that is the wrong way round; we should first consider how much non-human energy we really need to live satisfying lives, first), we need to investigate the environmental impact of each participant in that mix. We’ve ignored the environment for too long.

    • We have to do something, no matter what. The ONLY thing that could hurt the environment any more than what we are already collectively doing is to dig up the tar sands… and go into complete system collapse.
      Before reading “Do the math”, I was bent of “machine mass produced solar and batteries” (for pennies on the dollar)… as the solution.
      However, many people are actually against a global fleet powered by batteries and the hundreds of thousands of square kilometers of whatever the best machine made solar panels are!
      Now, I realize that we don’t need batteries, just electric cable or mini rail (like personal rapid transit).
      We don’t need all those solar panels either!
      But, in exchange, we need some advanced nuclear, preferably something like LFTR that is about 100 times more efficient (and much less dangerous) than today’s water and solid fuel based inefficient reactors.
      Currently, humanity consumes Billions of tons of FF’s and tens of thousands of tons of uranium every year. LFTR can do all that with just about 5,000 tons of thorium… The wastes problem is minuscule compared to today’s light water reactor, but there still is wastes. However, that waste (being only about 1/100th) lasts only 1/500th the time of conventional “spent wastes”.
      Now that’s a definite reduction of wastes by many orders of magnitude!

  32. Malthus will ultimately be proven right that growth collides with finite resources so that growth must stop.

    Malthus argued that population growth could never stop voluntarily because….contraception was immoral(!!). He has been proven wrong – just look at Japan and Italy.

    • Among other things. I would not go so far as to say that I think Malthus will be proven right on anything he mused about: just that growth will collide with finite resources and we lose that fight.

      • I’d stand up for that dismal scientist and rephrase you: He might have been wrong in every aspect – and often disgustingly so – except one, but that one is the one that really matters.

        I don’t have to be Irish to know man’s limitations. But then, Malthus is known – and loathed – more than the British bureaucrats that used his musings as a pretext to doing nothing during one of the better known die-offs in human history. The reason for that is that he identified one of the most terrifying issues we are dealing with, and nobody has ever come up with a solution. If we had all the oil, and none of the carbon to deal with, we’d still have to do some numbers – using some cruel equations – to find out whether there are already too many of us for a future with sustainability and equality. Limits are showing everywhere, these days.

        • Most environmentalists believe that the planet can not support 10 billion people.
          But the planet doesn’t have to.

          “My solution” is to mine the rare earths elements.
          This would not cause anywhere near the damage of mountaintop removal, global warming, future fracking and deforestation (and from tar sands removal).
          Use the rare earths for batteries, electric motors, etc, and use uranium and thorium in molten salt reactors for energy required to build and power up an electrical grid based transpo system (without batteries). This way, we still have cheap fossil fuel left to keep industry going for a much longer time.
          I would much rather trade “large, bulky and high speed” rail for “small, efficient and… more!” rail.

          With that, Malthus’ concerns are pushed farther into the future (again).

          • Confined to the narrow topic of energy, maybe. But what about soil erosion, desertification, deforestation, fisheries collapse, climate change (even if we stop fossil fuel cold turkey today—which we of course will not), species extinction, growth of dead zones, etc. Simply adopting nuclear energy in a big way may actually make these things worse and not better. So careful about the cavalier, simple answers!

          • I respect that you are a scientist, so I am kinda afraid to argue…
            It seems humanity should be able to use that abundant almost clean energy to address some of these problems. But then again I am being hopeful. At least, it would slow global warming.
            I “think” that fossil fuels are our worst enemy, that they are causing most of the problems. But, I have to admit that I don’t understand what really is causing desertification and soil erosion.

            I really wanted the hundreds of thousands of sq km of solar, but I’m afraid that would also cause problems and be much more difficult to implement because of the capacity and storage issues.

            Today, I’m even more riled up against fossil fuels… gas prices went up 5% since the last time I checked!

  33. Slightly off topic, but I’d love to see a numbers-wise treatment of ZEBRA/molten-salt thermal batteries as a part of the solution to the distributed storage problem. They seem well-suited to at least a home battery backup. A more in-depth treatment of the more speculative storage options, maybe?

  34. “How many computer records, tapes, CDROMs, etc., risk destruction or degradation in a collapse — even if it lasts only a century.” The Fermi paradox should make you even more pessimistic. If the Great Filter http://en.wikipedia.org/wiki/Great_Filter is due to resource exhaustion, but recovery within a century were possible, we should have seen evidence of alien civilizations by now.

  35. Kurzweil & Co’s anticipations are of an emergent system that will shake off many of the limitations prevalent in its substrate like a wet dog shakes off water. It is to be remembered that since Homo sapiens is a part of the substrate, in some outcomes we could prove to be Homo saps. Our significance in a post-singularity world may be comparable to apes on ours. 

  36. Hi Tom, you mention in one of your profile pages that you have implemented a range of changes to your house/lifestyle to stringly reduce your energy use. Can you elaborate on any of those to help others see what is practical and do-able within your own personal sphere. Not necessarily as a post but perhaps an additional page here on your site. E.g. solar HW, solar PV, insulation etc.

    cheers

  37. I am reminded of Booker T. Washington’s speech:

    “A ship lost at sea for many days suddenly sighted a friendly vessel. From the mast of the unfortunate vessel was seen a signal: “Water, water. We die of thirst.” The answer from the friendly vessel at once came back: “Cast down your bucket where you are.” A second time, the signal, “Water, send us water!” went up from the distressed vessel. And was answered: “Cast down your bucket where you are.” A third and fourth signal for water was answered: “Cast down your bucket where you are.” The captain of the distressed vessel, at last heeding the injunction, cast down his bucket and it came up full of fresh, sparkling water from the mouth of the Amazon River.”

    You say we can’t obtain liquid fuels to keep going the way we do? Well then, let’s not. Instead of turning rubber wheels on top of pavement, we can turn steel wheels on steel tracks. Immediately our dependency on liquid fuels is reduced. Then we electrify those tracks with overhead catenaries. And that need for liquid fuels goes away completely.

  38. In psychology there’s something called the Kübler-Ross model, commonly known as the Five Stages of Grief. It’s used to describe the process by which people deal with catastrophic loss.
    I have found the model also describes peoples’ reactions to bad news, such as peak oil.

    The stages include:

    Denial — “I feel fine.”; “This can’t be happening, not to me.”
    Denial is usually only a temporary defense for the individual. This feeling is generally replaced with heightened awareness of possessions and individuals that will be left behind after death.
    Anger — “Why me? It’s not fair!”; “How can this happen to me?”; ‘”Who is to blame?”
    Once in the second stage, the individual recognizes that denial cannot continue. Because of anger, the person is very difficult to care for due to misplaced feelings of rage and envy.
    Bargaining — “I’ll do anything for a few more years.”; “I will give my life savings if…”
    The third stage involves the hope that the individual can somehow postpone or delay death. Usually, the negotiation for an extended life is made with a higher power in exchange for a reformed lifestyle. Psychologically, the individual is saying, “I understand I will die, but if I could just do something to buy more time…”
    Depression — “I’m so sad, why bother with anything?”; “I’m going to die soon so what’s the point… What’s the point?”; “I miss my loved one, why go on?”
    During the fourth stage, the dying person begins to understand the certainty of death. Because of this, the individual may become silent, refuse visitors and spend much of the time crying and grieving. This process allows the dying person to disconnect from things of love and affection. It is not recommended to attempt to cheer up an individual who is in this stage. It is an important time for grieving that must be processed.
    Acceptance — “It’s going to be okay.”; “I can’t fight it, I may as well prepare for it.”
    In this last stage, individuals begin to come to terms with their mortality, or that of a loved one, or other tragic event.

    • I have accepted that we are going to lose liquid fuels as we know them. I have not quite accepted the outcome as we don’t know if electric mini rail will be allowed to be sufficient.

      However, we need to also accept the fact that money and economics will never be the same as machines (and robotics) will displace more jobs than they could ever create.
      We need to consider these two apparently catastrophic events together and in the right order to properly plan out the best possible future.
      I believe that our only hope of survival is for a unified call to action to enact whatever positive attributes necessary as outlined in a “best agreed upon social and economic plan based upon available, non Earth threatening actions and resources”.

    • My problem is that coming to terms with it means accepting that I can do nothing, and using the energy available now to do whatever I want to do, while I can. But if we all do that, the situation gets even worse…

  39. I thought I would post this recent animated documentary from the Post Carbon Institute that summarizes what this blog and others like it have been explaining — the whole problem laid out from A to Z. It comes to a pessimistic conclusion about our chances, a little too harsh I think about the opportunities that alternative energy technologies present, but otherwise pretty accurate.

    http://www.youtube.com/watch?v=VOMWzjrRiBg

  40. I’ve been reading this blog since I was linked to the Galactic Scale Energy post, which was a great way of demonstrating that it’s physically impossible to continue our growth on a finite planet. I’m a big fan of Dave McKay’s book and I’ve read what I can on how past societies have handled resource crunches – Jared Diamond’s books have some great examples.

    I’m also a scientist, and this post really brought home to me why the energy crisis bothers me so much. It’s that prospect that everything my colleagues and I are working on could simply disappear in less than a century. And even more frustratingly, that we could solve our energy problems with the resources and technology we have now, yet due to our political and social structures, we choose not to.

    I cycle, recycle, vermicompost, run solar panels, buy local, seasonal food, don’t use AC and don’t own a car. But our household is completely atypical for the neighbourhood, and even among my (educated and rational) colleagues, comfort and convenience take precedence over caring for the environment.

    It’s not a technological problem. It’s a social problem. How do we solve it? When I think about why I care about the issues, it’s because of my aforementioned worry that everything we have built will be destroyed, when it need not be. How do we
    1) convince people that the risk is real?
    2) convince them that they should do something about it? – especially when they may well be surrounded by people who have decided that they absolutely cannot change their lifestyle – until they are forced to.

    This presupposes that caring about consequences is a trait that most people possess. If people knew the consequences, would they change their behaviour? (i.e. are people just uneducated about the reality of the situation?) Or will most people refuse to change no matter how dire the consequences? It seems there are parallels with pro- and anti-smoking campaigns. Perhaps examining that social history would give us an idea of what kinds of incentives are most effective at changing behaviour on large scales. If it boils down to only economic incentives, then I doubt we will be able to avoid a crash. Does anyone know the largest example of a grouping of people sensibly managing a shared resource, and how they made that system effective?

    • Thanks for the thoughtful comment.

      I think many people would make useful and smart decisions if it could be unambiguously shown that our current path leads to collapse and great suffering. But I can’t prove this. No one really can. Uncertainty freezes out action. Yet for me, the uncertainty suggests playing it safe. Maybe other people would have a similar reaction if a convincing case could be made that we run significant risk of collapse on our current course. What is the level of significance that would lead to radical changes in behavior? 10%? 50%? 80%?. I don’t know.

      • That we can’t prove it does of course leave room for hope that we might be wrong. However, indefinite growth in consumption of a finite resource will result in exhaustion of that resource. To me, that’s a convincing enough argument straight away – the real crash might be three centuries down the line rather than three decades, but it’s inevitable unless consumption is stopped, or the resource is renewed (at a rate faster than depletion). Is it the uncertainty about when that freezes out action? People will act to save themselves, and their children, but not some nebulously defined future people who really ought to be able to look after themselves? Temporal tribalism…

        A note on the level of significance – it’s worth considering that people don’t make those judgements alone, but in the context of the behaviour of everyone around them (again, e.g. smokers). I wonder how we can best inform society in a way that causes peer pressure to push with us, rather than against us.

        • I’m with you on the joyous upside of being wrong. Let it be so.

          In terms of 30 vs. 300 years, we have enough obvious trouble coping with our current scale that I vote we slow down and take stock before deciding to barrel ahead. Treat it like an experiment: can humanity fashion a steady existence? Let’s test the brakes and see if we have control of the vehicle. How naive am I, to think “humanity” can agree on any course of action. We are merely victims to our nature, and will probably muddle along until we can’t any more.

          • We live in a time that every issue is left vs right, rural vs city and us vs them. Millions of people truly believe that Obama is ruining America and that global warming is a fraud so ‘lefties’ can force hard-working Americans to give up their livelihoods. And China and India will burn it all anyway. And you know what? Everyone’s right as much as anyone’s wrong. China and India might burn it anyway, and there is too much conflicting information these days for the average person to really make sense of global warming or Obama or anything else. We can’t build sustainable power because it’s too expensive and intermittent and, lets face it, just stupid in the light of cheap alternatives. We can’t tax carbon or make the cheap alternatives expensive, because it’s unfair and regressive. We can’t force people to drive less or implement road tolls or ask people to move closer to work. None of that will happen. Even among those that believe all of it, will we ever come to an agreement on a course of action that actually costs some of us money out-of-pocket?

            Let me plant another seed: whatever change we propose, it has to be cheap or free (in actual dollars, to actual people) otherwise it will never get past the Great Wall of Politics.

            We need a system to provide economic incentives to gradually reduce carbon use without being unfair to any individuals. It’s very close to impossible.

            Maybe we find out how much energy each person uses, and transfer their current income tax to an energy tax on a $/kilojoule basis.

            We do it for individuals as well as companies. You, Tom, might have to pay $1,000 tax per kilojoule while a Texas rancher might pay $.02 per kilojoule. So what?

            It’s the only way to be fair I can come up with because it’s revenue and cost neutral, China and India have no effect, rural vs urban and left vs right, no-one benefits more than anyone else.

            Collecting the energy-use data will be a nightmare. Just another thought experiment.

            My point is that whatever we (as an informed group) come up with, we can’t single out anyone to pay more than anyone else.

            Scott

          • Scott: actually it’s quite simple.
            http://en.wikipedia.org/wiki/Fee_and_dividend

            Raise carbon taxes to affect the price, give people money — starting with the collected tax — with which to pay the higher prices. But if they use less than average, they’ll in effect be paid rent by all the people polluting more than average. You’ll face $7/gallon gas but have money with which to pay for it, but if you can avoid using the gas, you pocket the money.

          • Damien:

            So people that use more than average will suddenly see their taxes and bills rise dramatically? People like farmers? Just won’t work, won’t pass in congress, will be shouted down. Sorry.

          • Dividend could be “topped up” to cover the worst cases not just the average. Fee could rise gradually rather than being slammed on in one year. Or businesses can rise the prices they sell at to cover their costs, which prices would be covered by the customer dividends. There are solutions.

            Or we could be so individualistic and divided that we let everyone block anything from happening and drag the country or even world into disaster unless other countries sanction us into responsibility. Having sane societal responses to environmental problems is not impossible in general, because societies do it. If the US can’t, then we’re self-doomed.

      • You’re formulating the problem the wrong way around. It’s not the level of significance but the timescale.
        The problem is not the _uncertainty_ about what will happen in the next 40 years. The problem is the 99.9% _certainty_ that nothing too bad will happen in the next 4 years (aka election period).
        Even if we knew with 100% certainty that there will be complete collapse in 40years, policy-makers still wouldn’t act until about the year 36, possibly later.

    • > Does anyone know the largest example of a grouping of people sensibly managing a shared resource, and how they made that system effective?

      You mention Jared Diamond. In “Collapse: How Societies Choose to Fail or Succeed,” Diamond examines numerous societies that (in his evidence-drenched estimation) chose to succeed. Arguably, the most relevant to post-industrial civilization is Edo-era Japan.

      On October 21, 1600, Tokugawa Ieyasu unified the Japanese Shogunate at the Battle of Sekigahara. Thus ushered in the Edo jidai, the longest contiguous period of sustained peace — both between otherwise divisive clans and between otherwise ecologically exploitative clans and the environment — in Japanese history. Indeed, Edo remains perhaps the best harbinger of a path forward for post-industrial civilization.

      Japan was, is, and probably always will be a communally legalistic patriarchy. By contrast, the United States was (as of its inception) and still is (but probably will cease to be, whether for good or ill) an individualistically legalistic patriarchy. While both systems of societal organization share superficial similarities, the leap from individualism to communalism is wider than most Americans might admit. Indeed, America’s dogmatic enshrinement of family-scale property rights to the exclusion of systemic-scale environmental rights may preclude humane and equitable solutions to our systemic problem of collapse.

      Then we should not expect pre-industrial Japanese solutions to scale to post-industrial America. Nonetheless, the Japanese solution is (to my knowledge) the only working example of steady state economics at the civilizational scale of labor division, societal stratification, literacy-driven technological resilience, surplus-driven agricultural resilience, and so forth. (Certainly, there exist working examples of steady state economics at the tribal scale. But those hardly apply.)

      How, pray tell, did the Tokugawa bakufu safeguard large-scale sustainability for over 450 years? Simple. State bureaucrats ruthlessly and laboriously accounted for every living tree in Japan, adjudicating swift and often harsh penalty for unlawful deforestation.

      While simple, effective, and seemingly essential, Ieyasu’s solution remains untenable in North America. Needless to say, I am not sanguine on our collective future.

    • “Does anyone know the largest example of a grouping of people sensibly managing a shared resource, and how they made that system effective?”

      I know but you wouldn’t like it.
      Stalin in Russia, managed to industialize the country in 20 years preparing in to the 2-nd World War, killing in the process about 10 millions. He took bread from starving people and sold it to the western countries to buy the technology. According to his records he was well aware of what he was doing.

      • Convince me that the Gulag Archipelago satisfies “a grouping of people sensibly managing a shared resource.” By “sensibly,” I’m fairly certain Ashwalker meant to imply “sustainably.”

        Clearly, the Former Soviet Union epitomized sustainability.

        • Surely it is sustainable, it solves the main problem of modern civilization – stops population growth. Moreover the use of resources per person is very small, and the required technological level is so low, that such a society can easily transition in the Middle Ages.

  41. At Risk of uttering the ultimate heresy, isn’t the real problem Capitalism? Many of the “best and brightest” (and most aggressive) members of our society are committed to a world view based on dramatically “dumbing down” the world’s complexity and shortening the time frame considered. That is, after all, what it means to think in terms of the “bottom line”. (Obviously,scientists are a different type of cat.)

    And what is “capitalism” but “rule by the people who own capital”? And who has the greatest vested interest in the status quo but the people who own a large share of lt? Even on the personal level I’ve noticed that people are most resistant to innovation and change when contemplating their greatest capital investments—home and children.

    • I think a lot of people are starting to realize the capitalism is no longer working (for us). The coming global debt crises will cause the wealthy to impose the golden rule, “those with the gold makes the rules”.

      Resources can vanish, not only because of physical limits, but because of distributional limits caused by economic breakdown.
      We have the tech necessary to prevent collapse, just not the proper social and monetary structure (I believe we should already have hundreds of thousands of sq km of machine mass produced solar arrays and batteries, electric mini rail AND advanced nuclear as a replacement for fossil fuels).

      After decline, can we work for free just to keep the infrastructure maintained and work towards building an economy based upon strategic elements, sustainable resources and efficiency? 🙂
      I hope that declining liquid fuels… and the wealthy do not declare war on such solutions!

    • How I see the political problem is that currently have a polarized left versus right circus that is devoid of any meaningful discussion of what centralized control and market forces can offer and what their respective drawbacks are. Certain political ideologies have been elevated to the status of religion, complete with armies of devoted followers. It’s become absurd.

      On one side we have the die hard free marketeers quick to point out how communism always fails. Therefore, the argument goes, we should have completely free markets (see Ludwig von Mises) since, of course, the world can be boiled down to a one dimensional continuum from left to right, and therefore there are no other alternatives… wink wink

      On the other hand we have somewhat more open minded people like scientists etc. pointing out that free markets simply cannot solve these problems either. They tend to be branded as “socialists” and vilified as communists, as if there’s something wrong with analyzing things at the social scale.

      So if communism clearly doesn’t work, and if free market capitalism doesn’t work, then shouldn’t we be talking about combining the good of each polarized ideology, weeding out the bad, and coming to some kind of a happy medium ground?

      But I think that level of discussion may be beyond what today’s juvenile political climate could entertain.

      • > On one side we have the die hard free marketeers quick to point out how communism always fails.

        China.

        > So if communism clearly doesn’t work…

        China.

        We may not agree with the authoritarian solution they’ve imposed, but can hardly dispute the evidence: planned market economies (either communist or socialist) with sufficient resources clearly outperform unplanned market economies (i.e., capitalist) with the same resources.

        As a self-identified anarchist, I would certainly prefer a locally decentralized economy. Unfortunately, human sapience (at least in North America) appears insufficient to support such a mass re-envisioning.

        • If you’re comparing China now to the US or other rich countries now, or even to early US development, that’s an invalid comparison. Developing countries today have the benefit of being able to copy existing technology and knowledge, while the fully developed countries had to develop it, and have to develop any further extensions now.

          I do believe government policies have their role, but you need more subtlety to build the case than just China’s growth rate.

          • ALL of the developed countries today have a history of copying/stealing ideas and technology from other countries. It’s not relevent in discussing what kind of resource allocation system we should employ.

        • It seems all may not be well in China.

          http://www.chrismartenson.com/blog/gordon-chang-reasons-chinas-imminent-bust/65040

          They are susceptible to the same bubbles we are and theirs may be worse. China is in an interesting situation, racing down the track bouncing between the guardrails of high inflation (if the central bank eases monetary policy to help with unemployment) on one side, and the other of high unemployment (the bank allows the Yuan to rise in value, thereby reducing foreign demand for China’s manufactured products).

          China would not have been able to achieve its tremendous growth if it hadn’t stolen America’s manufacturing base via currency manipulation (on the part of both China and the Fed). As a culture they are not good innovators; they are good copiers, which is one thing the US still has going for it.
          Plus China isn’t really a communist country. It has embraced a capitalist economy, and is seeing all of the predictable problems as a result.

          To sum up the problem as I see it in my post above: we are currently in a hybrid centrally planned / free market system. But it is no longer a democratic one serving the people. We are suffering from all the drawbacks from both central planning (manipulation of the monetary system by central banks to steal wealth) and free markets (wealth concentration, environmental and social externalities) — but enjoying none of the benefits of either. It’s therefore easy to find ammunition to defend any political stance one wants to take, since the current system is falling apart from all sides — that’s what Peak Oil means!

  42. Why to assume the conventional availability of resources as a limit to growth – might it be something as religious as taking granted expansion to space?
    Might it be more problematic to actually grow – what indications we have, that X size of population does not have other, earlier, more or less unknown limits to actual growth in numbers? Like limits inherent in species genetic similarities, in culture or individual psycology – the fear of limited resources as a one example. Reaction to that fear and draconian measures to cull resource use or population prematurely might be a significant limit itself.

    • A preponderance of ecologists estimate Homo sapiens to be at anywhere from 25% to 50% overshoot. Assuming a conservative underestimate of 25% overshoot, a sustainable human population not exceeding global carrying capacity would measure about 6.5 billion humans. Assuming the likelier underestimate of 50% overshoot, the same population would measure about 4.75 billion humans.

      In either case, we’ve substantially exceeded both measures. More growth only magnifies the gap between long-term necessity and short-term cultural inertia.

      See Mathis Wackernagel, et al.’s seminal paper “Tracking the ecological overshoot of the human economy” or the ecological motherload of all books, William R. Catton’s “Overshoot: The ecological basis of revolutionary change.”

      • Currently, it’s not the amount of people that matter, just the amount of all the different kinds of resources that we people consume due to politics and the capitalist social structure.
        I believe the “short term cultural inertia” is the major problem, both in actual inefficiency and in its fervent desire to keep the status quo.

  43. Very impressive bit of work you’ve done here. I share your trepidation about peak oil, the potential train wreck for humanity, decline of civilization, etc. Perhaps too alarmist for some, but it’s hard to argue with the facts: 1) humans use (need) a lot of energy, 2) oil is starting to peak/decline, and 3) we don’t know how to replace it.

    Please read our web site which, among other things, describes synthetic liquid fuel production. Questions and criticisms are welcome. The FAQs are a good place to start. This is a good page too: http://www.fusionpowercorporation.com/proprietary-advantage

    • The Black Death killed over one in three Europeans in just a few years, and European civilization got a bit eccentric in its behavior for a while, understandably. But it did not go away.

      So I don’t worry that civilization will decline. It’s been around for ten thousand years and taken shock after shock and is always functioning somewhere. I worry that civilization wil become largely poor and viciously unjust, because that’s the way it’s been for most of those millennia. We had only just begun to see our way to a propserous and just civilization with the coming of the Industrial Revolution. Can we contiue in this positive direction without exponentially-soaring extraction of cheap energy from the groumd?

  44. Tom,

    Spidey-sense may be a great starting point for a new design, especially when one has considered a complex system for years. I’ve been concerned about energy use and population growth since encountering Al Bartlett’s “The Forgotten Fundamentals of the Energy Crisis” in the late 70’s. Regarding supply, my spidey-sense puts nuclear at the top.
    http://www.albartlett.org/articles/art_forgotten_fundamentals_overview.html

    The premise implied by your blog title reminds us to quantify such things.
    This morning, I read an article (below) which gives performance numbers for a real solar thermal plant with storage. Heliostat field is “two miles across,” and annual electrical energy production is 5 x 10^11 Whr. A 1000 MW(e) nuclear plant may operate at 90% capacity factor, so the annual electrical energy production is 9 x 10^8 W x 24 x 365 hr, or 8 x 10^12 Whr. So, to supplant the nuclear plant with a solar-thermal-storage-electric one would require about (4 x 8 x 10^12) / (5 x 10^11), or 64 square miles of desert land. About eight miles on a side, with 16 circular arrays of ten thousand heliostats, and each array surrounding a 540 foot tower. This may be a labor intensive proposition, with 160,000 “billboard-sized heliostats” exposed to dry, dusty desert air. Also, Las Vegas is located where there is a frequently clear sky; mirrors won’t work as often and well near a more typical, coastal city. It remains to be seen whether this ideally situated project will deliver on promised costs, but after having worked at a 1000 MW(e) nuclear powerplant, this spidey suggests nuclear.
    http://news.cnet.com/8301-11386_3-57373508-76/solar-tower-will-power-las-vegas-at-night/?tag=mncol;editorPicks

    • Article says cost is just under $1 billion, for 110 megawatts of regular output, or $10 billion/gigawatt. That’s even more expensive than the new nuclear plant, which was $14 billion for 2 gigawatts. Though as the article says, it’s something of a prototype.

      13.5 cents a kilowatt hour doesn’t sound tragic.

      • 110 MWe is peak output. Averaged over the course of a year, this solar thermal storage electric plant is specified to operate at some 60% capacity factor, as can be gleaned from the annual energy output specified in the article. Our nuclear plants routinely run at over 90% capacity factor, so the cost swings even further in favor of nuclear.
        http://www.ans.org/pubs/magazines/nn/docs/2007-5-3.pdf

        Sure, the cost for powering Las Vegas in this manner is not tragic, and the method is environmentally superior to burning coal. But if solar costs more than old-school nuclear even under a clear desert sky, the differential becomes compelling for advanced nuclear at coastal locations where the rest of us live.

  45. The Solution

    In our case, the best that we can do is to aggressively pursue the most promising energy technology available. I and others have put forth molten salt reactor technology as a leading candidate, as its foundations have been well researched and its probable feature set a good match for our needs. It promises to be safe, clean, efficient, scalable, affordable, and flexible. Liquid fuels can be efficiently synthesized by these high temperature reactors, and synthesis is the only viable carbon-free/carbon-neutral alternative we have if we wish to maintain some semblance of modern transportation.

    As a backup, we can certainly pursue one of the other Gen IV designs like the very high temperature gas reactor which shares some technology with the MSR. Indeed, numerous related technologies, like the super-critical CO2 Brayton cycle conversion system, are already being pursued. Are we ready to significantly step up our financial commitment? Will increased resources meaningfully impact development? We need to hear from the relevant researchers to know for sure, but surely focusing attention on this critical path will help matters.

    Convincing the public should not be difficult once the engineering and scientific community begin speaking with one voice on this matter. The anti-nuclear lobby is a paper tiger in the face of our energy needs, and any irrationality can be brought to submit to this reality- the public will support a sane and hopeful proposal. And we are going to need that hope going forward as things are likely to get somewhat worse before they get better. Having a good plan is important for keeping the country united.

    Here are some details about a MSR design laid out back in 2007:

    http://energyfromthorium.com/2007/08/23/the-modified-geometry-2-fluid-molten-salt-breeder/

    A Gas Fast Reactor concept:

    http://prod.sandia.gov/techlib/access-control.cgi/2011/112525.pdf

    A recent Sandia paper on the super-critical CO2 Brayton cycle:

    http://prod.sandia.gov/techlib/access-control.cgi/2010/100171.pdf

Comments are closed.